Примеры по математике 5 класс на умножение и деление: Примеры на умножение и деление натуральных чисел | Учебно-методический материал по алгебре (5 класс) на тему:

Содержание

Самостоятельные работы по математике за 5 класс к учебнику Виленкина Н.Я. за 1, 2, 3 и 4 четверти — РОСТОВСКИЙ ЦЕНТР ПОМОЩИ ДЕТЯМ № 7

Содержание

Самостоятельные работы по математике за 5 класс к учебнику Виленкина Н.Я. за 1, 2, 3 и 4 четверти

Дата публикации: .

Самостоятельные на темы: «Натуральные числа и их обозначения», «Сложение и вычитание натуральных чисел», «Сравнение натуральных чисел», «Отрезок, прямая, луч», «Умножение натуральных чисел», «Деление натуральных чисел», «Выражения и уравнения», «Квадрат и куб числа», «Окружность и круг», «Обыкновенные дроби», «Сравнение дробей» и др.

Некоторые понятия к учебному материалу.

1. Натуральные числа – используются для счета предметов в повседневной жизни.
2. Отрезок. Длина отрезка – расстояние между его крайними точками, концами. Обозначается заглавными латинскими буквами, например AB.
3. Шкала – специальная линейка с делениями (штрихами).
4. Единичный отрезок – отрезок с длиной равной единице.
5. Меньше и больше. Меньше, число, которое при счете называется раньше. Больше, число, которое при счете называется позже.
6. Слагаемые числа – числа, которые складываются.

7. Вычитание. Число из которого вычитают – это уменьшаемое. Число, которое вычитается – это вычитаемое. В итоге получаем разность.

Самостоятельная работа №1 (входная работа на повторение)

Вариант I.

1. Определение числа.

а) Определите натуральное число, которое следует за числом 699.
б) Определите натуральное число, которое на две единицы меньше числа 1001.
в) Определите натуральное число, которое на единицу больше числа 239 999.
г) Определите натуральное число, которое на единицу меньше числа 394 000.

2. Решите задачу.

В городском сквере посажено 340 деревьев. А в парке посажено 270 деревьев. На сколько деревьев больше в городском сквере, чем в парке?

3. Решите примеры.

а) 492 + 1 220 =б) 3 495 — 593 =
в) 5112 : 6 =г) 56 * 23 =

Вариант II.

1. Определение числа.

а) Определите натуральное число, которое следует за числом 879.

б) Определите натуральное число, которое на единицу меньше числа 2 000.
в) Определите натуральное число, которое на единицу больше числа 722 999.
г) Определите натуральное число, которое на единицу меньше числа 24 000.

2. Решите задачу.

Рыбаки за первую неделю поймали 395 кг рыбы, а за вторую неделю – 239 кг. На сколько кг было поймано меньше за вторую неделю, чем за первую?

3. Решите примеры.

а) 638 + 1 445 =б) 6 112 — 2 598 =
в) 2688 : 3 =г) 24 * 45 =

Вариант III.

1. Определение числа.

а) Определите натуральное число, которое следует перед числом 699.
б) Определите натуральное число, которое на единицу меньше числа 3 000.
в) Определите натуральное число, которое на единицу больше числа 28 999.
г) Определите натуральное число, которое на единицу меньше числа 12 000.

2. Решите задачу.

В огороде посадили 2 грядки помидор. С первой грядки было собрано 427 помидор, а со второй грядки – 311. На сколько меньше помидор было собрано со второй грядки, чем с первой?

3. Решите примеры.

а) 455 + 3 412=б) 5 332 — 593 =
в) 3648 : 8 =г) 29 * 41 =
Самостоятельная работа №2 на тему: «Натуральные числа и их обозначения»

Вариант I.

1. Запишите следующие числа 3 раза подряд и запишите полученное в результате число в виде словосочетания.

а) число 20;
б) число 49.

2. Представьте следующие словосочетания в числовом виде.

а) Шесть миллиардов пятьсот три тысяча семь.
б) На единицу больше чем пятьсот девять тысяч девятьсот девяносто девять.

3. Определите все возможные трехзначные числа, состоящие из следующих чисел (числа не должны повторяться).

a) 2, 3 и 7.
b) 4, 0 и 9.

Вариант II.

1. Запишите следующие числа 3 раза подряд и запишите полученное в результате число в виде словосочетания.

а) число 60;
б) число 38.

2. Представьте следующие словосочетания в числовом виде.

а) Восемь миллиардов триста одна тысяча три.
б) На единицу больше чем сто девять тысяч девятьсот девяносто девять.

3. Определите все возможные трехзначные числа, состоящие из следующих чисел (числа не должны повторяться).

a) 1, 3 и 9.
b) 2, 4 и 0.

Вариант III.

1. Запишите следующие числа 3 раза подряд и запишите полученное в результате число в виде словосочетания.

а) число 30;
б) число 58.

2. Представьте следующие словосочетания в числовом виде.

а) Два миллиарда шестьсот два миллиона триста.
б) На единицу больше чем семьсот пять тысяч девятьсот девяносто восемь.

3. Определите все возможные трехзначные числа, состоящие из следующих чисел (числа не должны повторяться).

a) 5, 2 и 8.
b) 1, 3 и 0.

Самостоятельная работа №3

Вариант I.

1. Переведите из одной единицы измерения в другую.

а) 8 дм 43 см = … смб) 5 км 549 м = … м
в) 7 см 18 мм = … ммг) 249 см =… дм … см

2. Начертите отрезок AB, равный 17 см 5 мм. Отметьте на нем точки C и D. AC равно 10 см 4 мм, CD равно 4 см 9 мм. Чему равна длина отрезка DB?

3. Решите задачу.

Перед домом построили забор. Забор держится на 18 столбах, расстояние между столбами составляет пять метров. Каково расстояние между шестым и четырнадцатым столбами?

4. Начертите четырехугольник ABCD. Отметьте точкой T середину стороны BC. Соедините точки B и D, А и T. Выпишите все многоугольники, которые образовались.

Вариант II.

1. Переведите из одной единицы измерения в другую.

а) 4 дм 23 см = … смб) 25 км 50 м = … м
в) 16 см 65 мм = … ммг) 456 см =… дм … см

2. Начертите отрезок AB, равный 15 см 4 мм, отметьте на нем точки C и D. AC равен 8 см 2 мм, CD равен 3 см 7 мм.

Чему равна длина отрезка DB?

3. Решите задачу.

Перед домом построили забор. Забор держится на 19 столбах, расстояние между столбами составляет 4 метра. Каково расстояние между третьим и восьмым столбами?

4. Начертите четырехугольник ABCD. Отметьте середину AB и поставьте точку N. Проведите отрезки DN и АС. Выпишите все многоугольники, которые образовались.

Вариант III.

1. Переведите из одной единицы измерения в другую.

а) 19 дм 5 см = … смб) 21 км 678 м = … м
в) 43 см 8 мм = … ммг) 503 см =… дм … см

2. Начертите отрезок AB, равный 13 см 2 мм, отметьте на нем точки C и D. AC равен 7 см 3 мм. CD равен 3 см 6 мм. Чему равна длина отрезка DB?

3. Решите задачу.

Перед домом построили забор. Забор держится на 16 столбах, расстояние между столбами составляет 3 метра. Каково расстояние между пятым и одиннадцатым столбами?

4. Начертите четырехугольник ABCD. Отметьте середину CD и поставьте точку М. Проведите отрезки BM и АС. Выпишите все многоугольники, которые образовались.

Самостоятельная работа №4 на тему: «Сравнение натуральных чисел»

Вариант I.

1. Сравните числа.

а) 3 485 660 … 3 458 660б) 303 559 … 330 559
в) 2 596 440 … 2 569 440г) 45 696 … 44 696

2. Представьте в виде двойного неравенства: 18 т 347 кг … 18 т 4 ц 59 кг … 18 568 кг.

Вариант II.

1. Сравните числа.

а) 34 686 887 … 34 868 887б) 3 855 … 3 585
в) 40 955 999 … 40 595 999г) 455 776 … 445 776

2. Представьте в виде двойного неравенства: 13 км 845 м … 14675 м … 13 км 845 м 3 дм.

Вариант III.

1. Сравните числа.

а) 678 881 … 687 881б) 782 223 … 728 223
в) 2 490 606 … 2 490 660г) 13 799 . .. 13 977

2. Представьте в виде двойного неравенства: 15 т 475 кг … 15657 кг … 157 ц 35 кг.


Самостоятельная работа №5 на тему: «Сложение и вычитание натуральных чисел»

Вариант I.

1. Выполните сложение.

а) 348 588 667 + 239 586 394 =б) 93 955 483 + 495 868 991 =
в) 23 394 596 + 5 697 345 =г) 3 949 532 + 405 669 =

2. Выполните вычитание.

а) 348 588 667 — 283 745 733 =б) 93 955 483 — 22 394 583 =
в) 23 394 596 — 192 485 =г) 3 949 532 — 4 348 483 =

3. Решите задачу.

Мастерская закупила 560 гаек. На ремонт первой машины потребовалось 203 гайки, а на ремонт второй машины – еще 293 гайки. Сколько гаек осталось в мастерской?

4. Решите задачу.

В концертном зале стояло 454 стула. Для проведения концерта принесли 123 новых стула, а после антракта – еще 13 стульев. Сколько всего стульев стало в концертном зале?

Вариант II.

1. Выполните сложение.

а) 3 484 558 + 9 499 834 =б) 93 955 483 + 394 585 665 =
в) 3 495 863 + 35 384 588 =г) 5 697 291 + 34 405 669 =

2. Выполните вычитание.

а) 4 856 342 — 3 495 384 =б) 283 495 864 — 232 485 965 =
в) 5 965 493 — 3 449 594 =г) 23 455 303 — 19 485 588 =

3. Решите задачу.

В рулоне было смотано 327 м ленты. В первый день использовали 103 м, а во второй день – ещё 205 м. Сколько метров осталось в рулоне?

4. Решите задачу.

В магазине находилось 4 т 150 кг сахара. В первый день привезли 340 кг сахара, а во второй день – еще 4 ц сахара. Сколько кг сахара стало в магазине?

Вариант III.

1. Выполните сложение.

а) 2 399 388 + 239 586 394 =б) 435 483 + 495 868 991 =
в) 34 567 784 + 13 412 345 =г) 6 563 544 + 23 876 554 =

2. Выполните вычитание.

а) 455 586 661 — 283 745 733 =б) 40 954 586 — 22 394 583 =
в) 495 568 222 — 448 568 338 =г) 3 949 532 — 2 349 588 =

3. Решите задачу.

В моток смотано 459 м провода. В первый день истратили 119 м, а на второй день – 239 м провода. Сколько метров провода осталось в мотке?

4. Решите задачу.

На складе находилось 3 т и 450 кг муки. В первый день привезли 560 кг, через неделю привезли еще 5 ц муки. Сколько кг муки стало на складе?

Самостоятельная работа №6

Вариант I.

1. Найдите значение выражения: ( а + 46 ) : ( b — 48 ), если а = 35 и b = 57.

2. Упростите выражения.

а) с + 239 — 93;
б) 485 — 483 + d.

3. Составьте уравнение для решения задачи и решите его.

Было задумано некоторое число. К нему прибавили число 194, а потом прибавили ещё число 110 и получили число 322. Какое число было задумано?

4. Решите уравнения. a) (305 — ( ( 45 + х ) — 32 ) + 96 = 223;
б) 38 + ( 69 — y ) + 74 = 172.

Вариант II.

1. Найдите значение выражения: ( а — 34 ) * ( b + 9 ), если а = 60 и b = 11.

2. Упростите выражения.

а) 594 — 69 — а;
б) 149 + b — 54.

3. Составьте уравнение для решения задачи и решите его.

Было задумано некоторое число. Из этого числа вычли число 424, а затем прибавили число 392. В итоге, получилось число 632. Какое число было задумано?

4. Решите уравнения.

a) 209 — ( ( 145 + х ) — 12 ) + 96 = 123;
б) 18 + ( 159 — y ) + 34 = 172.

Вариант III.

1. Найдите значение выражения: ( а — 68 ) : b + 2 339, если а = 92 и b = 8.

2. Упростите выражения.

а) с + 239 — 193;
б) 485 — d + 384.

3. Составьте уравнение для решения задачи и решите его.

Было задумано некоторое число. Из этого числа вычли число 209, а затем прибавили число 47. В итоге, получилось число 217. Какое число было задумано?

4. Решите уравнения.

a) ( 111 — ( 45 + х ) ) + 96 = 123;
б) 29 + ( 59 — y ) + 15 = 72.

После завершения второй четверти, учащиеся должны:
1. уметь умножать натуральные числа и использовать эти знания;
2. уметь производить деление натуральных чисел, в том числе и деление с остатком, и использовать эти навыки при решении задач;
3. знать распределительное свойство умножения, уметь применять это свойство при устных вычислениях и при решении задач;
4. знать, что такое возведение числа в степень. Понимать, что такое корень и куб числа;
5. понимать, что такое формула, и как производить вычисления по формуле.

Самостоятельная работа №7 на тему: «Действия с натуральными числами. Умножение»

Вариант I.

1. Выполните умножение.

а) 283 * 46 =б) 29 * 473 =в) 841 * 93 =г) 19 * 632 =
д) 570 * 340 =е) 930 * 730 =ж) 5100 * 360 =з) 560 * 230 =

2. Умножьте числа, используя наиболее удобный порядок действий.

а) 25 * 491 * 4 * 200 =
б) 4 * 324 * 25 * 300 =

3. Расположите уравнения в порядке убывания, не производя никаких действий.

35 * 34 =34 * 33 =34 * 36 =32 * 32 =

4. Решите задачу.

В двухэтажной школе всего 32 кабинета и в каждом кабинете по 12 парт. В трехэтажной школе 45 кабинетов и в каждом кабинете по 14 парт. Сколько всего парт необходимо городским школам, если в городе 8 двухэтажных и 5 трехэтажных школ?

Вариант II.

1. Выполните умножение.

а) 342 * 57 =б) 64 * 268 =в) 342 * 89 =г) 32 * 864 =
д) 920 * 560 =е) 470 * 990 =ж) 2300 * 630 =з) 430 * 540 =

2. Умножьте числа, используя наиболее удобный порядок действий.

а) 25 * 376 * 4 * 500 =
б) 4 * 265 * 25 * 200 =

3. Расположите уравнения в порядке убывания, не производя никаких действий.

85 * 84 =84 * 83 =84 * 86 =82 * 82 =

4. Решите задачу.

В поселке построено 18 домов. Из них 4 трехэтажных, 6 двухэтажных, остальные одноэтажные дома. В трехэтажных домах – 18 окон, в двухэтажных – 14 окон, в одноэтажных – 8 окон. Сколько окон необходимо для 4 таких же посёлков?

Вариант III.

1. Выполните умножение.

а) 563 * 24 =б) 32 * 441 =в) 324 * 87 =г) 23 * 728 =
д) 220 * 680 =е) 240 * 580 =ж) 7500 * 290 =з) 920 * 630 =

2. Умножьте числа, используя наиболее удобный порядок действий.

а) 25 * 376 * 4 * 300 =
б) 4 * 641 * 25 * 100 =

3. Расположите уравнения в порядке убывания, не производя никаких действий.

65 * 64 =64 * 63 =64 * 66 =62 * 62 =

4. Решите задачу.

В один мешок помещается 26 кг картофеля, или 34 кг муки, или 38 кг сахара. Сколько всего весит груз, если в машину погрузили 32 мешка картофеля, 38 мешков муки и 52 мешка сахара?

Самостоятельная работа №8 на тему: «Деление натуральных чисел»

Вариант I.

1. Выполните деление.

а) 475 860 : 5 =б) 8 412 : 4 =в) 492 000 000 : 1 000 =
г) 270 930 : 3 =д) 386 240 : 5 =е) 19 688 : 23 =

2. Решите уравнения.

а) X : 85 = 2 210б) 36 690 : Y = 10в) 792 : X = 4
г) 15 * ( 39 : X ) = 45д) Y : 42 = 168е) 65 065 : Y = 1 001

3. Решите задачу.

Фермеру необходимо вспахать поле размером 318500 м. За сколько дней он вспашет поле, если известно, что за день он может вспахать 45 500 м?

4. Остаток равен 18, неполное частное – 35 и делитель – 23. Найдите делимое.

Вариант II.

1. Выполните деление.

а) 489 560 : 5 =б) 36 690 : 3 =в) 657 000 : 1 000 =
г) 960 552 : 6 =д) 522 240 : 2 =е) 67 065 : 85 =

2. Решите уравнения.

а) X : 26 = 456б) 4 760 : Y = 85в) 792 : X = 8
г) 35 * ( 54 : X ) = 315д) Y : 3 = 3015е) 524 : Y = 131

3. Решите задачу.

Станок производит 1200 заготовок за 1 час. Сколько минут нужно машине, чтобы приготовить 48 000 заготовок?

4. Остаток равен 33, неполное частное – 41 и делитель – 25. Найдите делимое.

Вариант III.

1. Выполните деление.

а) 236 560 : 4 =б) 36 690 : 6 =в) 612 345 000 : 1 000 =
г) 960 440 : 8 =д) 678 350 : 2 =е) 31 464 : 69 =

2. Решите уравнения.

а) X : 25 = 14б) 1 820 : Y = 28в) 1 836 : X = 6
г) 52 * Y = 468д) Y : 3 = 7 659е) 1048 : Y = 131

3. Решите задачу.

Комбайн убирает 30 га пшеницы за 1 час. Сколько дней ему нужно, чтобы убрать площадь равную 1200 га, если в день он будет работать по 10 часов?

4. Остаток равен 24, неполное частное – 25 и делитель – 28. Найдите делимое.


Самостоятельная работа №9 на темы: «Выражения, уравнения и решение уравнений», «Квадрат и куб числа»

Вариант I.

1. Решите примеры.

а) 34 + ( 239 — 606 : 6 ) * 4 — 393 : 3 =
б) 152 =
в) 73 =
г) ( 14 + 7 )2 — ( 5 + 13 )2 + 287 =

2. Упростите выражение и найдите его значение при с=34: 47с + 34 — 58 + 12с — 58.

3. Решите уравнения.

а) 15 * х = 945
б) 3 * y — 45 = 44

4. Решите задачу.

Бабушка и внучка слепили 124 пельмени. Сколько пельменей слепили бабушка и сколько внучка, если бабушка лепила в 3 раза быстрее, чем внучка?

Вариант II.

1. Решите примеры.

а) 472 — ( 29 + 124 : 4 ) — 72 : 8 =
б) 182 =
в) 63 =
г) ( 5 + 27 )2 — ( 4 + 12 )2 — 64 =

2. Упростите выражение и найдите его значение при с=12: 19с + 57 — 58с + 29с — 38 + 5с.

3. Решите уравнения:

а) 15 * х = 180
б) 12 * y + 36 = 96

4. Решите задачу.

Инженер и студент отремонтировали 248 приборов. Инженер ремонтировал приборы в 3 раза быстрее, чем студент. Сколько приборов починил каждый?

Вариант III.

1. Решите примеры.

а) 365 + ( 299 — 342 : 2 ) * 5 — 687 : 3 =
б) 172 =
в) 83 =
г) ( 4 + 7 )2 — ( 5 + 23 )2 + 787 =

2. Упростите выражение и найдите его значение при с=12: 47 + 56с — 6с + 34 — 12с.

3. Решите уравнения.

а) 32 * х = 1280
б) 8 * y + 36 = 356

4. Решите задачу.

Портной и его ученик сшили 213 фартуков. Портной работал в 2 раза быстрее, чем его ученик. Сколько фартуков сшил портной, а сколько ученик?

Самостоятельная работа №10 на темы: «Окружность и круг».
«Обыкновенные дроби»

Вариант I.

1. Нарисуйте окружность с центром в точке X и радиусом 4 см 6 мм. Нарисуйте отрезок CD так, чтобы он проходил через центр окружности и пересекал ее в точках C и D. Как называются отрезки СX и СD? Определите их длину.

2. Решите задачу.

Оля нашла 26 грибов, из них 18 маслят. Какую часть грибов составляют маслята?

3. Решите задачу.

Рыбаки поймали 112 кг рыбы. Из них 1028 – караси. Сколько карасей поймали рыбаки?

4. Решите задачу.

Коля прочитал 85 страниц журнала, что составило 512 от общего числа страниц. Сколько страниц в журнале?

Вариант II.

1. Нарисуйте окружность с центром в точке Y и радиусом 3 см 8 мм. Нарисуйте отрезок EF так, чтобы он проходил через центр окружности и пересекал ее в точках E и F. Как называются отрезки YE и EF? Определите их длину.

2. Решите задачу.

Коля собрал в корзину 31 фрукт, из них 22 фрукта – это груши. Какую часть собранных фруктов составляют груши?

3. Решите задачу.

Школьники собрали 104 кг овощей. 1326 от общего числа овощей составляют помидоры. Сколько кг помидор собрали школьники?

4. Решите задачу.

Мастер отремонтировал 35 приборов, что составило 512 от общего количества приборов. Сколько всего приборов надо отремонтировать мастеру?

Вариант III.

1. Нарисуйте окружность с центром в точке Z и радиусом 2 см 6 мм. Нарисуйте отрезок GH так, чтобы он проходил через центр окружности и пересекал ее в точках G и H. Как называются отрезки GZ и GH? Определите их длину.

2. Решите задачу.

У Саши есть 29 карандашей. Из них 19 карандашей – это простые карандаши. Какую часть карандашей составляют цветные карандаши?

3. Решите задачу.

Мастер сделал 312 деталей. Из них 324 часть деталей – деревянные. Сколько деревянных деталей сделал мастер?

4. Решите задачу.

Ребята из 5 класса собрали 32 кг ягод. Это составляет 324 от всего количества собранных ягод. Сколько всего ягод было собрано?

Самостоятельная работа №11 на тему: «Сравнение дробей»

Вариант I.

1. Задан луч длиной в 12 единиц. Отметьте на числовом луче:

а) 212 частиб) 612 части23 части54 части

2. Сравните дроби.

а) 2338 и 1618

б) 2145 и 1526

3. Найдите три решения неравенства.

а) 2122< x < 2222

б) 711 < z < 811

4. При каких значениях х:

а) дробь х22 будет правильной?

б) дробь 15х будет неправильной?

Вариант II.

1. Задан луч длиной в 15 единиц. Отметьте на числовом луче:

415 части315 части35 части23 части

2. Сравните дроби.

а) 2634 и 1517

б) 2249 и 1821

3. Найдите три решения неравенства.

а) 1920 < x < 2020

б) 79 < z < 89

4. При каких значениях y:

а) дробь y19 будет правильной?

б) дробь 23y будет неправильной?

Вариант III.

1. Задан луч длиной в 18 единиц. Отметьте на числовом луче:

218 части618 части23 части56 части

2. Сравните дроби.

а) 2631 и 1819

б) 2341 и 1718

3. Найдите три решения неравенства.

а) 910< y < 1010

б) 57 < z < 67

4. При каких значениях z:

а) дробь z29 будет правильной?

б) дробь 13z будет неправильной?

Самостоятельная работа №12 на тему: «Сложение и вычитание обыкновенных дробей»

Вариант I.

1. Решите примеры.

а) 2631 + 1831631;

б) 171255125 + 106125;

в) 1939 + ( 1839639 ) — 1339;

2. Решите уравнения.

а) x + 618 = 1618

б) 1325 — ( y + 625 ) = 425

3. Решите задачу.

Первый спортсмен пробежал 57 км, а второй спортсмен за тоже время пробежал 67 км. На сколько метров больше пробежал первый спортсмен?

4. Решите задачу.

Из мешка взяли 29 части муки, а потом – ещё 39 части. В мешке осталось 14 кг. Сколько кг муки было в мешке?

Вариант II.

1. Решите примеры.

а) 1538 + 12381138;

б) 231928192 + 48192;

в) 1956 + ( 21561256 ) — 1656;

2. Решите уравнения.

а) x — 512 = 312

б) 1823 — ( 723 + y ) = 523

3. Решите задачу.

Расстояние от дачи до пруда равно 35 км, а от дачи до леса равно 45 км. На сколько метров расстояние от дачи до пруда больше, чем расстояние от дачи до леса?

4. Решите задачу.

Из погреба вытащили 312 части картофеля, а потом – ещё 212 части. После этого в погребе осталось 56 кг картофеля. Сколько картофеля было в погребе?

Вариант III.

1. Решите примеры.

а) 1928 + 12281628;

б) 1317611176 + 49176;

в) 2742 + ( 1242642 ) — 1242;

2. Решите уравнения.

а) x + 1223 = 2023

б) 2835 — ( y + 1635 ) = 435

3. Решите задачу.

Расстояние от школы до больницы равно 89 км, а от школы до бассейна равно 49 км. На сколько метров расстояние от школы до больницы больше, чем расстояние от школы до бассейна?

4. Решите задачу.

Из рулона отрезали 38 части ткани, а потом – ещё 28 части. После этого в рулоне осталось 32 метра ткани. Сколько метров ткани было в рулоне?

Самостоятельная работа №13 на тему: «Сложение и вычитание смешанных чисел»

Вариант I.

1. Решите примеры.

а) 4 1928 + 6 1228;

б) 5 13176 — 2 11176;

в) 12 2743 + 3 1243.

2. Решите уравнения.

а) 23 1838 + х =36 1228;

б) 7 1416 — y = 3 1116;

в) y + 18 2753 = 24 1353;

3. Решите задачу.

В первый день в мастерской использовали 23 318 метра проволоки, а во второй день – ещё 18 218 части. После этого в рулоне осталось 32 метра проволоки. Сколько метров проволоки было в рулоне?

Вариант II.

1. Решите примеры.

а) 3 1322 + 3 1222;

б) 8 15126 — 4 15126;

в) 13 2249 + 3 1449.

2. Решите уравнения.

а) 2 1843 + х = 3 443;

б) 17 1519 — y = 12 1219;

в) y — 18 3856 = 24 2756.

3. Решите задачу.

В первый день в школе покрасили 17 523 метра коридора, а во второй день – ещё 23 423 метра. Сколько метров было покрашено за 2 дня?

Вариант III.

1. Решите примеры.

а) 5 1923 + 6 1223;

б) 7 1348 — 3 1148;

в) 82 2578 + 34 1278

2. Решите уравнения.

а) 6 1729 + х = 23 429;

б) 8 15128 — y = 6 12128;

в) y — 18 3847 = 5 2747.

3. Решите задачу.

Фермер убрал 13 613 метра грядки в первый день, а на следующий день – ещё 18 313 метра. После двух дней работы осталось убрать 6 метров. Какова длина грядки?

Самостоятельная работа №14 на темы: «Десятичная запись дробных чисел». «Сравнение десятичных дробей»

Вариант I.

1. Заданные дроби представьте, как десятичные дроби.

а) 5 5910
б) 6 1100

в) 17 1371000

2. Сравните числа.

а) 5,596 и 5,629
б) 7,34 и 7,339
в) 0,684 и 0,6840

3. Переведите из одной единицы измерения в другую.

а) представьте в тоннах: 92 ц; 887 кг; 14 т 12 кг;
б) представьте в квадратных дециметрах: 8 м 2; 57 см 2; 8 м2 77 дм2.

4. Отметьте точки: 0,2; 0,8; 1,1; 2,3; 2,1; 3,7 на числовом отрезке, равном 5 единицам.

Вариант II.

1. Заданные дроби представьте, как десятичные дроби.

а) 18 591000

б) 710

в) 7 137100

2. Сравните числа.

а) 35,97 и 35,971
б) 8,449 и 8,540
в) 0,92 и 0,920

3. Переведите из одной единицы измерения в другую.

а) представьте в тоннах: 3 ц; 239 кг; 23 т 28 кг;
б) представьте в квадратных дециметрах: 13 м 2; 2 см 2; 87 м2 32 дм2.

4. Отметьте точки: 0,5; 0,7; 1,1; 2; 2,3; 3,5 на числовом отрезке, равном 6 единицам.

Вариант III.

1. Заданные дроби представьте, как десятичные дроби.

а) 15 43100

б) 9 231000

в) 510

2. Сравните числа.

а) 29,345 и 29,354
б) 171,89 и 171,889
в) 0,93 и 0,930

3. Переведите из одной единицы измерения в другую.

а) представьте в тоннах: 18 ц; 56 кг; 3 т 9 кг;
б) представьте в квадратных дециметрах: 4 м 2; 23 см 2; 2 м2 56 дм2.

4. Отметьте точки: 0,4; 0,5; 1,4; 1,9; 2,4; 3,0 на числовом отрезке, равном 4 единицам.


Самостоятельная работа №15 на темы: «Сложение и вычитание десятичных дробей». «Округление чисел»

Вариант I.

1. Решите примеры на сложение десятичных дробей.

а) 29,3 + 4,35 =
б) 68,9 + 19,1 =
в) 0,68 + 6,4 =

2. Решите примеры на вычитание десятичных дробей.

а) 35,1 — 13,2 =
б) 37 — 27,3 =
в) 13,28 — 5,327 =

3. Решите задачу:

В первый день плот проплыл 14,8 км, во второй день – на 1 км 700 м больше, чем в первый день. В третий день плот проплыл на 600 м меньше, чем во второй день. Сколько всего км проплыл плот?

4. Округлите:

а) целую часть числа 2539,48190 до сотен, до десятков, до единиц;
б) дробную часть числа 2539,48190 до тысячных, до сотен, до десятков.

Вариант II.

1. Решите примеры на сложение десятичных дробей.

а) 79,3 + 8,15 =
б) 18 + 8,8 =
в) 0,93 + 23,4 =

2. Решите примеры на вычитание десятичных дробей.

а) 48,2 — 4,98 =
б) 96 — 48,6 =
в) 37,67 — 13,168 =

3. Решите задачу.

В первом пакете было 15,7 кг песка, во втором – на 350 г больше, чем в первом. В третьем – на 1200 г меньше, чем в первом. Сколько кг песка в трех пакетах?

4. Округлите:

а) целую часть числа 3462,9470 до сотен, до десятков, до единиц;
б) дробную часть числа 3462,9470 до тысячных, до сотен, до десятков.

Вариант III.

1. Решите примеры на сложение десятичных дробей.

а) 34,3 + 13,11 =
б) 8 + 47,7 =
в) 0,123 + 23,942 =

2. Решите примеры на вычитание десятичных дробей.

а) 69,2 — 7,88 =
б) 91,76 — 18,6 =
в) 8,94 — 5,452 =

3. Решите задачу.

3 дня бабушка пекла блины. В первый день она использовала 1,2 кг муки, во второй день – на 500 г меньше, чем в первый день, а на третий день – на 300 г больше, чем во второй день. Сколько муки она использовала за три дня?

4. Округлите:

а) целую часть числа 4392,73910 до сотен, до десятков, до единиц;
б) дробную часть числа 4392,73910 до тысячных, до сотен, до десятков.

Самостоятельная работа №16 на тему: «Умножение десятичных дробей на натуральные числа»

Вариант I.

1. Выполните умножение.

а) 8,3 * 8 =б) 7,12 * 34 =в) 0,235 * 93 =г) 1,93 * 100 =

2. Найдите значение выражения: х + ( 3,74х — 1,474х ) при х=3; 100; 374; 1000.

3. Решите задачу.

Одновременно навстречу друг другу из двух деревень, расстояние между которыми составляет 45,8 км, вышли пешеходы. Скорость первого пешехода составляет 4,2 км/ч, а скорость второго – 4,5 км/ч. Какое расстояние будет между ними через 4 часа?

4. Решите задачу.

Машина проехала 360 км за 6 часов. Какое расстояние она преодолеет, передвигаясь с той же скоростью, за 14 часа, за 2 13 часа?

Вариант II.

1. Выполните умножение.

а) 7,48 * 12 =б) 3,57 * 7 =в) 0,873 * 87 =г) 1,698 * 1000 =

2. Найдите значение выражения: 5х + ( 6,59х + 2,483х ) при х=5; 100; 324; 1000.

3. Решите задачу.

Одновременно в противоположных направлениях из города выехали 2 машины. Скорость первой машины составляет 54,7 км/ч, а скорость второй – 76,2 км/ч. Какое расстояние будет между ними через 3 часа?

4. Решите задачу.

Велосипедист преодолел 72 км за 3 часа. Какое расстояние он преодолеет, перемещаясь с той же скоростью, за 56 часа, за 2 13 часа?

Вариант III.

1. Выполните умножение.

а) 9,4 * 6 =б) 8,34 * 56 =в) 0,517 * 62 =г) 6,787 * 1000 =

2. Найдите значение выражения: ( 8,45х — 3,594х ) — х при х=8; 100; 843; 1000.

3. Решите задачу.

Одновременно навстречу друг другу из двух городов выехали мотоциклы. Расстояние между городами составляет 234,8 км. Скорость первого мотоциклиста составляет 34,5 км/ч, а скорость второго – 56,2 км/ч. Какое расстояние будет между ними через 2 часа?

4. Решите задачу.

Моторная лодка прошла 24 км за 2 часа. Какое расстояние она пройдет, перемещаясь с той же скоростью, за 14 часа, за 3 13 часа?

Самостоятельная работа №17 на тему: «Деление десятичных дробей на натуральные числа»

Вариант I.

1. Выполните деление.

а) 2,729 : 6 =б) 283,85 : 4 =в) 4 : 13 =г) 0,095 : 10 =

2. Решите уравнения.

а) 5X — 0,4 = 23,6б) 48,2 : Y = 10,4

3. Решите задачу.

За два дня рабочие отремонтировали 3,6 км дороги. В первый день они отремонтировали 1/4 части дороги. Сколько км дороги они отремонтировали во второй день?

4. Решите задачу.

4 класс и 5 класс собирали макулатуру. Пятиклассники собрали в 2 раза больше макулатуры, чем ребята из 4 класса. Вместе они собрали 239,7 кг. Сколько кг собрали ребята из 5 класса и сколько ребята из 4 класса?

Вариант II.

1. Выполните деление.

а) 5,837 : 7 =б) 291,49 : 5 =в) 5 : 18 =г) 0,023 : 10 =

2. Решите уравнения.

а) 8X + 2,8 = 18,6б) 28,1 : Y = 12,4

3. Решите задачу.

За два дня бригада собрала 147,6 кг ягод. В первый день они собрали 4/9 части урожая ягод. Сколько кг ягод они собрали во второй день?

4. Решите задачу.

Две бригады собирали картофель. Первая бригада собрала в 3 раза больше картофеля, чем вторая. Обе бригады вместе собрали 49,6 ц урожая. Сколько центнеров картофеля собрали первая бригада и сколько вторая бригада?

Вариант III.

1. Выполните деление.

а) 4,752 : 9 =б) 472,49 : 6 =в) 7 : 19 =г) 0,044 : 10 =

2. Решите уравнения.

а) 5X + 2,5 = 24б) 14,2 : Y = 3,4

3. Решите задачу.

За 2 дня мотоциклист преодолел 394,1 км. В первый день он проехал 47 части пути. Сколько км он проехал во второй день?

4. Решите задачу.

Мама собрала в 5 раз больше ягод, чем дочка. Вместе они собрали 34,5 кг ягод. Сколько ягод собрала мама и сколько дочка?

Самостоятельная работа №18 на тему: «Среднее арифметическое»

Вариант I.

1. Найдите среднее арифметическое четырех чисел: 4,5; 5,6; 4,9; 5,1.

2. Решите задачу.

В течение часа машина двигалась со скоростью 67,5 км/ч, в течение второго часа – со скоростью 51,6 км/ч. В течение третьего часа её скорость составила 72,3 км/ч. Какова средняя скорость машины? Сколько км она преодолела за 3 часа?

3. Решите задачу.

Среднее арифметическое трех чисел составляет 14,5. Первое число – 14,1, а второе число на 0,8 больше третьего числа. Назовите эти числа.

4. Решите задачу.

Расстояние между двумя деревнями равно 340 км. Автомобиль преодолел половину пути со скоростью 58 км/ч, а вторую половину – со скоростью 49 км/ч. Какова средняя скорость автомобиля на протяжении всего пути?

Вариант II.

1. Найдите среднее арифметическое четырех чисел: 12,3; 12,9; 11,6; 13,1.

2. Решите задачу.

В течение первого часа спортсмен шел со скоростью 11,2 км/ч, в течение второго часа – со скоростью 10,7 км/ч, а в течение третьего часа его скорость составила 9,8 км/ч. Какова средняя скорость спортсмена? Какое расстояние он прошел за 3 часа?

3. Решите задачу.

Среднее арифметическое трех чисел составляет 28,5. Первое число – 28,2, а второе на 0,9 больше третьего числа. Назовите эти числа.

4. Решите задачу.

Расстояние между двумя городами составляет 52 км. Первую половину пути велосипедист передвигался со скоростью 18 км/ч, а вторую половину – со скоростью 22 км/час. Какова средняя скорость велосипедиста на всем протяжении пути?

Вариант III.

1. Найдите среднее арифметическое четырех чисел: 9,1; 9,9; 11,1; 10,7.

2. Решите задачу.

В течение первого часа лодка двигалась со скоростью 15,5 км/ч, во второй час движения её скорость составила 17,4 км/ч, а в течение третьего часа – 12,7 км/ч. Какая средняя скорость лодки? Сколько км она преодолела за 3 часа?

3. Решите задачу.

Среднее арифметическое трех чисел составляет 13,2. Первое число – 13,9, а второе – на 0,7 больше третьего числа. Назовите эти числа.

4. Решите задачу.

Расстояние между двумя деревнями составляет 24 км. Первую половину пути пешеход двигался со скоростью 8 км/ч, а вторую половину – со скоростью 9 км/ч. Какова средняя скорость пешехода на всем протяжении пути?

Самостоятельная работа №19 на тему: «Проценты, задачи на проценты»

Вариант I.

1. Решите задачу.

В спортивной секции занимается 60 учеников, из них 70% составляют девочки. Сколько мальчиков занимается в спортивной секции?

2. Решите задачу.

Ребята четвертых и пятых классов собирали макулатуру. Ребята пятого класса собрали 150 кг макулатуры, что составило 60% общего веса собранной макулатуры. Сколько кг макулатуры собрали ребята?

3. Решите задачу.

Из 15 кг яблок получается 12 кг яблочного пюре. Каков процент выхода пюре из яблок?

Вариант II.

1. Решите задачу.

В 5 классе числится 30 учеников, 60% из них – мальчики. Сколько девочек учится в 5 классе?

2. Решите задачу.

2 бригады собирали помидоры. Первая бригада собрала 320 кг помидор, что составило 40% от общего урожая. Сколько всего помидор собрали обе бригады?

3. Решите задачу.

Из 60 семян взошли 55 растений. Найдите процент всхожести семян.

Вариант III.

1. Решите задачу.

В школе работает 40 человека. Из них 80% – женщины. Сколько мужчин работает в школе?

2. Решите задачу.

Бабушка и внучка собирали яблоки. Бабушка собрала 30 кг яблок, что составило 80% от общего сбора. Сколько кг яблок собрали бабушка и внучка вместе?

3. Решите задачу.

При перемалывании 40 кг зерна получили 25 кг муки. Найдите процент выхода муки.

Математика 5 класс — Мир математики

 I.  Чтобы разделить десятичную дробь на натуральное число, нужно делить дробь на это число, как делят натуральные числа и поставить в частном запятую тогда, когда закончится деление целой части.

Примеры. 

Выполнить деление1) 96,25:5;  2) 4,78:4;  3) 183,06:45.

Решение.

Пример 1) 96,25:5.

Делим «уголком» так, как делят натуральные числа. После того, как сносим цифру 2 (число десятых — первая цифра после запятой в записи делимого 96,25), в частном ставим запятую и продолжаем деление.

Ответ19,25.

 

 

Пример 2) 4,78:4.

Делим так, как делят натуральные числа. В частном поставим запятую сразу, как снесем 7 — первую цифру после запятой в делимом 4,78. Продолжаем деление дальше. При вычитании 38-36 получаем 2, но деление не окончено. Как поступаем? Мы знаем, что в конце десятичной дроби можно приписывать нули — от этого значение дроби не изменится. Приписываем нуль и делим 20 на 4. Получаем 5 — деление окончено.

Ответ1,195.

 

Пример 3) 183,06:45.

Делим как 18306 на 45. В частном поставим запятую как только снесем цифру 0 — первую цифру после запятой в делимом 183,06. Так же, как в примере 2) нам пришлось приписать нуль к числу 36 — разности чисел 306 и 270.

Ответ: 4,068.

Вывод: при делении десятичной дроби на натуральное число в частном ставим запятую сразу после того, как сносим цифру в разряде десятых делимого. Обратите внимание: все выделенные красным цветом цифры в этих трех примерах относятся к разряду десятых долей делимого.

Смотрите видео: «Как разделить десятичную дробь на натуральное число».

II. Чтобы разделить десятичную дробь на 10, 100, 1000 и т. д. нужно перенести запятую влево на 1, 2, 3 и т. д. цифр.

Примеры.

Выполнить деление: 1) 41,56:10;  2) 123,45:100;  3) 0,47:100;  4) 8,5:1000;  5) 631,2:10000.

Решение.

Перенос запятой влево зависит от того, сколько в делителе нулей после единицы. Так, при делении десятичной дроби на 10мы будем переносить в делимом запятую влево на одну цифру; при делении на100 — перенесем запятую влево на двецифры; при делении на 1000 перенесем  в данной десятичной дроби запятую на три цифры влево. 

В примерах 3) и 4) пришлось приписать нули перед десятичной дробью, чтобы удобнее было переносить запятую. Однако, приписывать нули можно мысленно, и вы будете это делать, когда хорошо научитесь применять правило II для деления десятичной дроби на 10, 100, 1000 и т. д.

Смотрите видео: «Как разделить десятичную дробь на 10, 100, 1000 и т.д.»

 

I. Чтобы умножить десятичную дробь на натуральное число, нужно умножить ее на это число, не обращая внимания на запятую, и в полученном произведении отделить запятой столько цифр справа, сколько их было после запятой в данной дроби.

Примеры. Выполнить умножение: 1) 1,25·7;   2) 0,345·8;   3) 2,391·14.

Решение.

Смотрите видео: « Как умножить десятичную дробь на натуральное число».

II. Чтобы умножить одну десятичную дробь на другую, нужно выполнить умножение , не обращая внимания на запятые, и в полученном результате отделить запятой справа столько цифр, сколько их было после запятых в обоих множителях вместе.

Примеры. Выполнить умножение: 1) 18, 2·0,09;   2) 3,2·0,065;    3) 0,54·12,3.

Решение. 

Смотрите видео: «Умножение десятичных дробей.»

III. Чтобы умножить десятичную дробь на 10, 100, 1000 и т. д. нужно перенести запятую вправо на 1, 2, 3 и т. д. цифр.

Примеры. Выполнить умножение: 1) 3,25·10; 2) 0,637·100; 3) 4,307·1000; 4) 2,04·1000; 5) 0,00031·10000.

Решение.

Смотрите видео: «Умножение десятичных дробей на 10, 100, 1000 и т. д.»

IV. Чтобы умножить десятичную дробь на 0,1; 0,01; 0,001 и т. д. нужно перенести запятую влево на 1, 2, 3 и т. д. цифр. 

Примеры. Выполнить умножение: 1) 28,3·0,1; 2) 324,7·0,01; 3) 6,85·0,01; 4) 6179,5·0,001;  5) 92,1·0,0001.

Решение.

Смотрите видео: «Умножение десятичных дробей на 0,1; 0,001; 0,0001 и т. д.»

  Поиск Поиск

  • Школьный помощник
    • математика 5 класс
    • математика 6 класс
    • алгебра 7 класс
    • алгебра 8 класс
    • геометрия 7 класс
    • русский язык 5 класс
    • русский язык 6 класс
    • русский язык 7 класс
  • математика
  • алгебра
  • геометрия
  • русский язык

«»

следующая предыдущая вернуться на предыдущую страницу

Такой страницы нет !!!

  • Популярные запросы
    • Обстоятельство
    • Дополнение
    • Определение
    • Деление дробей
    • Русский язык 7 класс
    • Русский язык 6 класс
    • Русский язык 5 класс
    • Алгебра 7 класс
    • Математика 6 класс
    • Алгебра 8 класс
    • Математика 5 класс
    • Наименьшее общее кратное
    • Наибольший общий делитель. Взаимно простые числа
    • Буквы о и а в корнях -кос- / -кас-; -гор- / — гар-; -клан- / -клон-; -зар- / -зор-
    • Буквы о и а в корнях -кос- / -кас-; -гор- / — гар-; -клан- / -клон-; -зар- / -зор-
    • Деление и дроби
    • Квадратный корень из неотрицательного числа
    • Доли. Обыкновенные дроби
    • Окружность и круг
    • Антонимы. Синонимы
    • Десятичная запись дробных чисел
    • Буквы о – а в корнях -лаг- / -лож-, -рос- / -раст- (-ращ-)

Примеры на умножение и деление натуральных чисел | Учебно-методический материал по алгебре (5 класс) на тему:

«Мастер-умножитель»

Предлагаются занимательные примеры. При решении каждого из них получается слово (для этого используем ключ).

                

1) Ключ         0   1   2   3   4   5   6    7   8   9  

                       И   ф  л   р   н   а   м   о   г   ь

689832 : 67                888982 : 43                181804 : 604

928564 : 91                595161 : 987        965280 : 32

147368 : 169                550940 : 65

767010 : 74                686868 : 546

                

2) Ключ            0   1   2   3   4   5   6   7   8   9

                                    ь   о   л   с    т   п   а   к   р   е

778 ∙ 73                                1398 ∙ 25                102 ∙ 88

25945 ∙ 16                        23458 ∙ 37                21333 ∙ 37

166 ∙ 49                         83 ∙ 32                 4102 ∙ 97

1023 ∙ 71                        5489 ∙ 14

                        

3) Ключ                0   1   2   3   4   5   6   7   8   9

                                к   с   о   в   а   т    е   и   н   р

976968 : 27                        947648 : 64                101188 : 82

956662 : 23                        638638 : 77                657072 : 18

                

4) Ключ                        0   1   2   3   4   5   6   7   8   9

                                        ь   л   п   е   в   о   н   к   м   а

128520 : 17                        238280 : 28                87653 : 37

9972 : 12                        73160 : 31                99374 : 22        

96930 : 45                        151656 : 71

                

5) Ключ                        0   1   2   3   4   5   6   7   8   9

                                        ь   к   у   р   п   а   н   о   л   т

4127 ∙ 111                        367 ∙ 34                2026 ∙ 76

3133 ∙ 155                        28711 ∙ 16                883 ∙ 15

365 ∙ 131                        452 ∙ 15                3367 ∙ 53

751 ∙ 165                        1149 ∙ 85

1097 ∙ 443                        10084 ∙ 45

328 ∙ 283                          485 ∙ 17

6) Ключ                         0   1   2   3   4   5   6   7   8   9

                                        о   м   л   ч  п   я   с   и   а    е

49110 : 321                        967296 : 352                815670 : 19

81832 : 212                        612469 : 143                511225 : 715

178192 : 344                        865260 : 23

                        

7) Ключ                        0   1   2   3   4   5   6   7   8   9

                                        а   х   м   б  н   и   о   к   е   л        

276504 : 984                        922507 : 23                196317 : 99

954434 : 26                                984256 : 14                778088 : 19        

972928 : 32                                962090 : 47

205407 : 787                        826386 : 209

                        

8) Ключ                        0   1   2   3   4   5   6   7   8   9

                                л   о   с   п   р   а   к   е    т   в

2591 ∙ 111                        373 ∙ 244                2179 ∙ 28

1954 ∙ 175                        142 ∙ 197                755 ∙ 47

773 ∙ 385                        134 ∙ 187                508 ∙ 69

2113 ∙ 432                        713 ∙ 905                375 ∙ 827        

1477 ∙ 145                        1993 ∙ 305

                

Задачи и примеры по математике за 5 класс: тренажер по математике для 5 класса онлайн

Ваш ребенок с трудом решает задачи и не может освоить десятичные дроби? Мы поможем решить эту проблему. Предлагаем пройти тест по математике за 5 класс на интеллектуальной платформе Skills4u абсолютно бесплатно. Всего за несколько минут вы сможете оценить уровень владения материалом и получите рейтинг, составленный на основе правильных ответов.

Наша платформа предлагает интерактивные тесты по математике (5 класс), основанные на интеллектуальном алгоритме. Суть в том, что выдача заданий происходит на основе анализа результатов опроса. Это персонализированный подход, позволяющий учесть уровень подготовки каждого ученика и предложить примеры, которые ему по силам. При этом каждый раз задания могут быть иными, постепенно усложняясь по мере усвоения материала.

Посещает ли ваш ребенок частную или государственную школу, занимается дополнительно онлайн или ходит к репетитору – тестирование по математике за 5 класс выявит пробелы в знаниях и поможет сформировать прочные навыки решения любых задач. Весь материал разбит по темам. Вы можете выбрать ту, в которой ваш ребенок испытывает затруднения, или сделать полный срез успеваемости, чтобы составить полную картину. Пробный онлайн тест по математике за 5 класс вы можете пройти совершенно бесплатно прямо сейчас. Платформа оценит ответы, предложит исправить ошибки и составит рейтинг ученика.

Для формирования устойчивого навыка, доходящего до автоматизма, необходимо решать примеры по вычислительным навыкам за 5 класс по математике в течение нескольких последующих дней для закрепления материала. Для этого следует оформить доступ к образовательной платформе Skills4u, выбрав один из планов: на 1 месяц, на полгода или на 12 месяцев. Стоимость невелика, а результат будет очень скоро заметен.

Если ваш ребенок будет регулярно проходить онлайн тестирование за 5 класс по математике на интерактивной платформе, он сможет быстро и безошибочно решать самые сложные примеры и уравнения, производить вычисления с десятичными дробями. Разумеется, гарантированный положительный результат дает только продолжительная подписка, но даже месяц занятий позволит сдвинуться с мертвой точки.

Увлекательная форма без дополнительных письменных заданий, интеллектуальный алгоритм выдачи примеров, позволяющий учитывать уровень подготовки каждого ребенка – вот залог успеха нашей программы. Присоединяйтесь!

Урок 14. деление нацело — Математика — 5 класс

Математика

5 класс

Урок № 14

Деление нацело

Перечень вопросов, рассматриваемых в теме:

— деление натуральных чисел;

— свойства деления натуральных чисел.

Тезаурус

Деление – это математическое действие, обратное умножению.

Делимое – это число, которое делят.

Делитель – это число, на которое делят.

Частное – результат деления.

Делить на нуль нельзя.

Любое натуральное число а делится на 1 и само на себя:

а : 1 = а, а : а = 1

Важное свойство частного: делимое и делитель можно одновременно умножить или разделить на одно и то же натуральное число: частное от этого не изменится.

Обязательная литература

  1. Никольский С. М. Математика: 5 класс. // С. М. Никольский, М. К. Потапов, Н. Н. Решетников, А. В. Шевкин. – М.: Просвещение, 2017. – 272 с.
  2. Потапов М. К. Математика. Книга для учителя. 5-6 классы. // М. К. Потапов, А. В. Шевкин. – М.: Просвещение, 2010.- 256 с.

Дополнительная литература

  1. Бурмистрова Т. А. Математика. Сборник рабочих программ. 5-6 классы. // Составитель Т. А. Бурмистрова – М.: Просвещение, 2014.- 80 с.
  2. Потапов М. К. Математика: дидактические материалы. 6 класс. // М. К. Потапов, А. В. Шевкин – М.: Просвещение, 2010.- 118 с.
  3. Чесноков А. С. Дидактические материалы по математике 5 класс. // А. С. Чесноков, К. И. Нешков. – М.: Академкнига, 2014.- 124 с.

Теоретический материал для самостоятельного изучения

Давайте вспомним, что нам уже известно об операции деления. Пусть у нас есть натуральные числа a и b, причём а больше b или равно b (a ≥ b). Говорят, что а делится на b нацело, если существует натуральное число с, при умножении которого на b получается а: a = b ∙ c.

Обычно слово «нацело» в этой фразе опускается. При этом записывают: a : b = с и называют а – делимым, b – делителем, с – частным.

Любое натуральное число а делится на 1 и само на себя:

а : 1 = а, а : а = 1

так как а ∙ 1 = а, 1 ∙ а = а.

Например, 14 делится на 1 и на 14.

14 : 1 = 14, 14 : 14 = 1

При делении ноля на любое натуральное число получается ноль: 0 : а = 0, потому что 0 ∙ а = 0.

Запомните: делить на нуль нельзя!

Любое натуральное число а делить на нуль нельзя, потому что не существует такого числа с, для которого выполнялось бы равенство а : 0 = с (так как с ∙ 0 = 0 ≠ а). Принято считать, что нуль на нуль делить нельзя.

Для деления чисел из двух и более цифр (знаков) применяют деление уголком.

Вспомним, как делить уголком, на примере.

Вычислим 392 : 28 = ?

Для начала запишем делимое и делитель уголком:

Начнём делить 392 на 28 следующим образом.

Во-первых, определим неполное частное. Для этого слева направо сравниваем цифры делимого и делителя.

Рассмотрим цифру 3. Она меньше 28 – значит, нужно взять ещё одну цифру из делимого. 39 больше 28, следовательно, это неполное частное.

Ставим точку в частном (под уголком делителя).

Посчитаем, сколько цифр осталось в делимом, после неполного частного. У нас после 39 стоит только одна цифра – 2. Значит, и в результат добавляем ещё одну точку.

Приступаем к делению: 28 помещается в 39 только один раз, поэтому ставим первой цифрой ответа единицу и вычитаем 28 из 39.

После вычитания в остатке получилось 11, это меньше, чем 28, поэтому к 11 дописываем 2.

112 делится на 28. Получаем 4. Записываем полученный результат второй цифрой в ответе.

В остатке получился нуль – значит, числа разделились нацело. Таким образом, 392 : 28 = 14.

Важное свойство частного: делимое и делитель можно одновременно умножить или разделить на одно и то же натуральное число: частное от этого не изменится.

Вычислим 50 : 25 = ?

Сначала одновременно умножим 50 и 25 на 2. Получим:

100 : 50 = 2.

Теперь разделим 50 и 25 на 5. Получим:

10 : 5 = 2.

В обоих случаях ответ оказался одинаковым. Значит, свойство частного верно.

Если каждое из натуральных чисел a и b делится на натуральное число с, то верно равенство:

(a+ b) : c = a : c + b : c.

Убедимся в правдивости данного свойства на примере.

Вычислим выражение: 124 : 4 + 36 : 4.

Рассмотрим два способа решения.

1 способ. Выполним деление и сложим результаты.

124 : 4 + 36 : 4 = 31 + 9 = 40.

2 способ. Заметим, что у нас есть общий делитель – 4. Вынесем его за скобки. Получим:

(124 + 36) : 4 = 160 : 4 = 40.

В обоих случаях у нас получился один и тот же ответ. Значит, свойство верно.

Разбор решения заданий тренировочного модуля

№ 1. Вычислите 812 : 14 = _____.

Решение: выполним деление уголком.

Ответ: 58.

№ 2. Найдите неизвестный множитель х из равенства: 15 ∙ х = 195.

Выберите верный ответ: х = 3; х = 13; х = 25; х = 15.

Решение: чтобы найти неизвестный множитель, надо произведение поделить на известный множитель, то есть:

15 ∙ х = 195

х = 195 : 15

Выполнив деление уголком, получим:

Ответ: х = 13.

Урок 17. деление с остатком — Математика — 5 класс

Математика

5 класс

Урок № 17

Деление с остатком

Перечень вопросов, рассматриваемых в теме:

— деление с остатком;

— неполное частное;

— остаток.

Тезаурус

Деление с остатком – это деление одного натурального числа на другое, при котором остаток меньше делителя.

Обязательная литература

  1. Никольский С. М. Математика: 5 класс. // С. М. Никольский, М. К. Потапов, Н. Н. Решетников, А. В. Шевкин. – М.: Просвещение, 2017. – 272 с.
  2. Потапов М. К. Математика. Книга для учителя. 5-6 классы. // М. К. Потапов, А. В. Шевкин. – М.: Просвещение, 2010.- 256 с.

Дополнительная литература

  1. Бурмистрова Т. А. Математика. Сборник рабочих программ. 5-6 классы. // Составитель Т. А. Бурмистрова – М.: Просвещение, 2014.- 80 с.
  2. Потапов М. К. Математика: дидактические материалы. 6 класс. // М. К. Потапов, А. В. Шевкин – М.: Просвещение, 2010.- 118 с.
  3. Чесноков А. С. Дидактические материалы по математике 5 класс. // А. С. Чесноков, К. И. Нешков. – М.: Академкнига, 2014.- 124 с.

Теоретический материал для самостоятельного изучения

Не всегда можно полностью разделить одно число на другое. В примерах на деление может оставаться остаток. Такое деление называется деление с остатком.

Рассмотрим пример. Разделим 16 на 5.

Запишем этот пример в столбик:

Получилось, что 5 помещается в 16 три раза, но остаётся 1 – это остаток.

Читается данное выражение следующим образом: «16 разделить на 5 получится 3, и остаток – 1».

Деление с остатком – это деление одного натурального числа на другое, при котором остаток меньше делителя.

Если при делении натуральных чисел остаток равен нулю, то договорились считать, что делимое делится на делитель без остатка, или делится нацело.

Запишем деление с остатком в общем виде.

Порядок решения выражений на деление с остатком:

1. находим наибольшее число до а, которое делится на b без остатка – это c;

2. вычитаем из делимого найденное число c.

a c = r

Сравниваем остаток с делителем. Остаток всегда меньше делителя: r < b.

Если получилось, что остаток больше делителя – значит, наибольшее число, которое делится на делитель без остатка, найдено неверно.

При решении более сложных примеров не всегда можно легко найти наибольшее число из пункта 1. Иногда для этого необходимо произвести дополнительные расчёты в столбик.

Рассмотрим ещё один пример.

297 : 25 = ?

Запишем это выражение в столбик:

Получили остаток 22, он меньше, чем 25, значит:

297 : 25 = 11 ост (22)

Как проверить деление с остатком:

  1. умножить неполное частное на делитель;
  2. прибавить к полученному результату остаток;
  3. сравнить полученный результат с делимым.

Проверим ответ предыдущего примера.

297 : 25 = 11 ост (22)

25 · 11 = 275

275 + 22 = 297

Деление с остатком выполнено верно.

Разбор решения заданий модуля

№ 1. Вычислите выражение 312 : 15 = _____ ост (____)

Решение: выполним деление уголком:

Сравним неполное частное с делителем: 12 < 15.

Теперь проверим, верно ли мы нашли неполное частное и остаток:

20 ∙ 15 + 12 = 300 + 12 = 312

Ответ: 312 : 15 = 20 ост (12)

№ 2. Найдите неизвестное делимое в выражении:

х : 17= 18 (остаток 4)

Выберите верный ответ: х = 310; х = 120; х = 250; х = 110.

Решение: чтобы найти неизвестное делимое, надо неполное частное умножить на делитель и прибавить остаток.

х = 18 ∙ 17 + 4

х = 306 + 4

х = 310

Ответ: х = 310.

Стратегии разделения для 5-го класса

Боитесь ли вы преподавать разделение каждый год? Раньше я был в этом лагере. Мои ученики много боролись с делением, потому что им не хватало чувства числа и концептуального понимания. Однако, к счастью, я понял, что так быть не должно. Я могу использовать стратегии разделения, которые не только подходят для развития, но и имеют смысл для моих учеников.

Фактически, как только я отказался от стандартного алгоритма в качестве моей стратегии деления, мои ученики начали осваивать деление влево и вправо.Некоторые все еще боролись? да. К сожалению, ни одна из стратегий или советов, которыми я делюсь, не станет волшебной пилюлей, но они сделают разделение доступным и концептуальным для ваших учеников.

Частные частные с помощью Easy Breezy Math

Частные частные изменили правила игры для моих учеников. Частные частные — это стратегия деления, которая использует фрагменты для более эффективного многократного вычитания групп делителя.


Но мне пришлось добавить эшафот для моих учеников.Мы называем этот эшафот нашей «Легкой и легкой математикой». По сути, мы берем известное нам умножение и списываем их в сторону. Мы устраняем эти проблемы, чтобы помочь нам решить проблему. Мы делаем это, потому что моим ученикам действительно не хватает чувства деления в начале года. Это также помогает им увидеть связь между умножением и делением.

По мере того, как год идет, они выйдут за рамки простой простой математики, поскольку они разовьют чувство числа и начнут видеть закономерности.

Совет: напомните им сложить множители вертикально в их Easy Breezy Math. Иногда я даю своим ученикам доски специально для этого, чтобы сделать их более интересными.

Чтобы получить более подробную информацию об этом методе разделения и получить помощь бесплатно, щелкните здесь.

Если эта стратегия для вас нова и вам нужно увидеть ее в действии, то посмотрите это видео и песню.

Коробочная модель

или коробочный метод с частичными частными

Box Model Division 1 был для меня новым годом назад и работает лучше всего, когда студенты знакомы с использованием этого типа органайзера при умножении на частичные произведения. Этот тип стратегии деления похож на частичное частное, но организован иначе.

Если это для вас совершенно в новинку, посмотрите видео, на котором учитель и ее класс проходят этапы. Этот метод лучше всего «увидеть в действии», а не читать о нем.

Это видео длиннее, но оно отлично помогает учителям понять стратегию деления и послушать некоторый язык, чтобы помочь ученикам понять его.Примечание. Часть работы What I Know совпадает с тем, что я называю « Easy Breezy Math.

Я настоятельно рекомендую представить эту стратегию (и действительно любую стратегию, но определенно эту) в контексте проблемы со словом, чтобы помочь студентам осмыслить стратегию и действительно понять, что происходит. Полезно называть дивиденд и делитель в зависимости от контекста.

Советы для преподавателей (с любой стратегией дивизиона)

1.) Представьте новые стратегии с увлекательным контекстом или проблемой со словом. По мере того, как вы проходите этапы стратегии, проходите через то, что может происходить в проблеме.

2.) Если эти стратегии совершенно новые, то потратьте день (или больше) с однозначными делителями, прежде чем переходить к двузначным делителям. Это будет того стоить!

3.) Убедитесь, что ученики понимают цель Easy Breezy Math, если вы решите использовать эту платформу. Кроме того, убедитесь, что они понимают, что это означает и как работает (в нем представлены частичные частные, которые уже готовы для учащихся на выбор).

4.) Научите студентов проверять свои ответы умножением. Это поможет им в самооценке и позволит им обнаруживать небольшие ошибки, которые они допускают при вычитании или другие ошибки вычислений.

А как насчет традиционного алгоритма деления?

Поскольку в моем штате используется общее ядро, я не преподаю традиционный алгоритм деления в рамках учебной программы пятого класса. Однако после государственного тестирования мы делаем предварительный просмотр математики в шестом классе. В то время я всегда усиленно учил традиционный алгоритм.К этому моменту у моих учеников есть четкое понимание деления и лучшее чувство чисел. Это помогает им освоить алгоритм. Кроме того, я обнаружил, что многие учителя шестого класса ожидают, что они знают традиционный алгоритм, поэтому я определенно не хочу, чтобы мои ученики отставали, когда они пойдут в среднюю школу.

Требуется БОЛЬШЕ ресурсов для подразделения?

БЕСПЛАТНЫЕ печатные формы «Дивизион»: Щелкните здесь, чтобы получить бесплатный пакет печатных материалов «Дивизион дня». Я использую их для утренней работы, проверки математики или домашних заданий после того, как ученики выучили деление и им нужно развить беглость речи и продолжить практику.

Дополнительные ресурсы отдела

: У меня также есть пакет отдела, который содержит печатные формы, простые центры, карточки задач и оценки. Ресурсы не требуют какой-либо конкретной стратегии, поэтому вы можете выбрать стратегии, которые вы хотите, чтобы ваши ученики применяли, или позволить им выбирать. Щелкните здесь, чтобы просмотреть дополнительные ресурсы Division в моем магазине Teachers Pay Teachers.

Используете ли вы какую-либо из этих стратегий разделения? Вы используете другую стратегию разделения? Дайте мне знать в комментариях.Я всегда ищу новые стратегии разделения для 5-го класса!

Рабочие листы

Division для 3, 4 и 5 классов

Вы здесь: Главная → Рабочие листы → Division

Это бесплатные распечатываемые рабочие листы по разделам, сгенерированные случайным образом для учащихся 3-5 классов. Темы включают факты деления, умственное деление, деление в столбик, деление с остатками, порядок операций, уравнения и факторинг.

Вы можете распечатать их прямо из окна браузера, но сначала проверьте, как это выглядит в «Предварительном просмотре».Если рабочий лист не помещается на странице в режиме предварительного просмотра, настройте поля, верхний и нижний колонтитулы в настройках страницы в браузере.

Или же, отрегулируйте «масштаб» до 90% или меньше в предварительном просмотре. В некоторых браузерах может быть опция «Печатать по размеру», которая автоматически масштабирует рабочий лист до такого размера, чтобы он соответствовал области печати.

Все рабочие листы имеют ключ ответа; однако вам нужно щелкнуть страницу ключа ответа сразу же, после создания рабочего листа, потому что ключ ответа также генерируется «на лету» и не будет существовать позже, если вы придете искать его позже.

Перейти к:


4 класс

Рабочие листы ментального деления

Практика деления фактов (таблицы 1-10)
Практика деления фактов (таблицы 1-12)
Отсутствует дивиденд или делитель (основные факты)
Разделить на 10 или 100
Разделить на целые десятки или сотни
Разделите в уме целые десятки и целые сотни на однозначные числа

Деление с остатком от 1 до 100 по основным фактам
Деление с остатком в пределах 1-100
Деление с остатком, делитель целых десяти

Деление с остатком, делителем на целую сотню

Порядок операций: сложение, вычитание, умножение, деление и скобка — три операции
Порядок операций: сложение, вычитание, умножение, деление и скобка — четыре операции
Порядок операций: сложение, вычитание, умножение, разделение и скобка — пять операций

Листы с длинным делением

Практика деления фактов (с использованием символа длинного деления)

Длинное деление, двузначные дивиденды, однозначный делитель, точное деление
Деление в столбик, двузначные дивиденды, однозначный делитель, возможны остатки

Длинное деление, трехзначные дивиденды, однозначный делитель, точное деление
Деление в столбик, трехзначные дивиденды, однозначный делитель, возможны остатки

Длинное деление, 4-значные дивиденды, 1-значные делители, точное деление
Длинное деление, 4-значные дивиденды, 1-значные делители, возможны остатки

5 класс

Психическое подразделение

Разделите в уме трех- и четырехзначное число на однозначное

Деление с остатком в пределах 1-100
Деление с остатком, делитель целых десяти
Деление с остатком, делитель на целую сотню

Деление в длину

1-значный делитель, 4-значное делимое, без остатка
1-значный делитель, 4-значное делимое, остаток

2-значный делитель, 4-значное делимое, делитель между 11 и 35
2-значный делитель, 4-значное делимое, без остатка — (делителем является любое двузначное число)
2-значный делитель, 4-значное делимое, с остатком — (делителем является любое двузначное число)

Умножение уравнения (пропущенный множитель; решить в столбик)
Уравнения деления (пропущенный делимый или делитель; решение путем длинного умножения или длинного деления)
Решите уравнения умножения — используя длинное деление, одно- или двухзначный делитель

Решите уравнения деления — используйте либо длинное деление, либо умножение


Таблицы факторинга

Перечислить все множители данного числа

Факторинговые числа в пределах 4-100 до простых множителей
Сложный факторинг: числа множителей в пределах 4-500 в простых множителях

Бесплатные задания по математике для 5-х классов

Вы здесь: Главная → Задания → 5 класс

Это исчерпывающий набор бесплатных распечатываемых рабочих листов по математике для 5 класса, организованных по таким темам, как сложение, вычитание, алгебраическое мышление, разряд, умножение, деление, разложение на простые множители, десятичные дроби, дроби, измерения, координатная сетка и геометрия. Они генерируются случайным образом, их можно распечатать в вашем браузере и включать в себя ключ ответа. Рабочие листы подходят для любой математической программы для пятого класса, но особенно хорошо подходят для программы IXL по математике для 5-го класса и их новых уроков внизу страницы.

Рабочие листы генерируются случайным образом каждый раз, когда вы нажимаете на ссылки ниже. Вы также можете получить новый, другой, просто обновив страницу в своем браузере (нажмите F5).

Вы можете распечатать их прямо из окна браузера, но сначала проверьте, как это выглядит в «Предварительном просмотре».Если рабочий лист не умещается на странице, отрегулируйте поля, верхний и нижний колонтитулы в настройках страницы вашего браузера. Другой вариант — настроить «масштаб» на 95% или 90% в предварительном просмотре печати. В некоторых браузерах и принтерах есть опция «Печатать по размеру», которая автоматически масштабирует рабочий лист по размеру области печати.

Все рабочие листы содержат ключ ответа, расположенный на 2-й странице файла.


Алгебра
Математика для начальных классов Эдвард Заккаро

Хорошая книга по решению проблем с очень разнообразными текстовыми задачами и стратегиями решения проблем.Включает главы по следующим темам: последовательности, решение проблем, деньги, проценты, алгебраическое мышление, отрицательные числа, логика, отношения, вероятность, измерения, дроби, деление. Вопросы в каждой главе разбиты на четыре уровня: легкий, несколько сложный, сложный и очень сложный.


Сложение и вычитание по столбцам (числа друг под другом)

Место и округление

Пропускной счет

  • Пропуск на 20 000, начиная с 550 000
  • Пропуск на 50 000, начиная с 120 000
  • Пропуск на 100 000, начиная с 1 350 000
  • Пропуск на 100 000, начиная с 628 000
  • Пропуск на 300 000, начиная с 4 250 000
  • Пропуск на 500 000, начиная с 750 000

Округление

  • Округлить до десяти в пределах от 0 до 10 000
  • Округлить до ближайшей сотни в пределах от 0 до 1 000 000
  • Округлить до ближайшей тысячи в пределах от 0 до 1 000 000

  • Смешанные задачи округления 1 — округление до ближайших десяти, сотен или тысяч
  • Смешанные задачи округления 2 — округление до ближайших десяти, сотен, тысяч или десяти тысяч
  • Смешанные задачи округления 3 — как указано выше, но округление до подчеркнутой цифры
  • Смешанные задачи округления 4 — округление до подчеркнутой цифры с округлением до ближайшего миллиона

Умножение

Умножение умственных способностей

Длинное умножение (в столбцах)


Подразделение

Психологическое отделение

Длинное деление

  • 1-значный делитель, 4-значное делимое, без остатка
  • 1-значный делитель, 4-значное делимое, остаток

  • 2-значный делитель, 4-значное делимое, делитель от 11 до 35
  • 2-значный делитель, 4-значное делимое, без остатка — (делителем является любое двузначное число)
  • 2-значный делитель, 4-значное делимое, с остатком — (делителем является любое двузначное число)

  • Умножение уравнения (пропущенный множитель; решить путем деления в столбик)
  • Уравнения деления (отсутствует делимое или делитель; решается путем умножения или деления в столбик)

Следующие четыре типа рабочих листов выходят за рамки Стандарт Common Core для пятого класса.

Факторинг

Дробное сложение и вычитание

Как дроби / дробные части

В отличие от дробей / дробных частей

  • Сложить или вычесть непохожие дроби — знаменатели 2, 3, 4, 5, 6, 8 и 10
  • Сложить или вычесть непохожие дроби — знаменатели 2-12
  • Задача: сложить или вычесть непохожие дроби — знаменатели 2-25
  • Задача: сложить или вычесть 3 в отличие от дробей — знаменатели 2, 3, 4, 5, 6, 8 и 10

  • Сложить или вычесть смешанные числа — знаменатели 2-12
  • Сложить или вычесть смешанные числа — знаменатели 2-25

  • Сложить или вычесть смешанное число и дробь или целое число — знаменатели 2-12
  • Сложить или вычесть смешанное число и дробь или целое число — знаменатели 2-25

Умножение на дробь

Фракционное подразделение

Следующие типы рабочих листов выходят за рамки стандартов Common Core.


Преобразование дробей в смешанные числа и наоборот

Эквивалентные дроби и упрощенные дроби
Записывать дроби как десятичные и наоборот

В приведенных ниже таблицах ключ ответа не дает дроби в упрощенной форме. Например, 0,24 задается как 24/100, а не как 6/25. Если хотите, вы можете спросить студент упростить.

Сложение десятичной дроби

Ментальная математика

От 0 до 1 десятичных цифр

От 0 до 2 десятичных цифр

Дополнительная колонка


Десятичное вычитание

Ментальная математика

От 0 до 1 десятичных цифр

От 0 до 2 десятичных цифр

Задачи: ментальная математика

Вычитание по столбцу

Вызовы: алгебраическое мышление


Десятичное умножение

Ментальная математика

  • Умножение целого числа на десятичное — просто (одна десятичная цифра)
  • Умножить целое число на десятичное — сложнее (одна десятичная цифра)
  • Умножение целого числа на десятичное — пропущенный множитель (одна десятичная цифра)

  • Умножить целое число и десятичная дробь (1-2 десятичные цифры)
  • Умножить целое число и десятичное число — пропущенный множитель (1-2 десятичных знака)

  • Умножение целого числа на десятичное (1-3 десятичных знака)
  • Умножение целого числа на десятичное — коэффициент пропущенного (1-3 десятичных знака)

  • Умножить десятичные дроби на десятичные
  • Умножение десятичных знаков на десятичные — пропущенный коэффициент

  • Умножение десятичных знаков на десятичные или целые числа (смешанная практика)
  • Умножение десятичных дробей на десятичные или целые числа — пропущенный коэффициент (смешанная практика)

  • Умножить на 10 или 100 (1-2 десятичных знака)
  • Умножить на 10, 100 или 1000 (1-2 десятичных знака)
  • Умножить на 10, 100 или 1000 — пропущенный коэффициент (1-2 десятичные цифры)

  • Умножить на 10 или 100 (1-3 десятичных знака)
  • Умножить на 10, 100 или 1000 (1-3 десятичных знака)
  • Умножить на 10, 100, 1000, 10000 или 100000 (1-3 десятичных знака)
  • Умножение десятичных знаков на 10, 100 или 1000 — отсутствующий множитель (1-3 десятичных знака)

Умножить по столбцам


Десятичное деление

Ментальная математика

Длинное деление


Единицы измерения

Обычная система

  • Конвертировать между дюймы и футы — проще
  • Преобразование между дюймами и футами — сложнее

  • Конвертировать между дюймы, футы и ярды — проще
  • Преобразование между дюймами, футами и ярдами — сложнее
  • Преобразование дюймов, футов и ярдов с десятичными знаками — используйте калькулятор

  • Преобразование миль, ярдов и футов 1 — с помощью калькулятора
  • Преобразование миль, ярдов и футов 2 — с помощью калькулятора

  • Преобразование между унциями и фунтами — проще
  • Преобразование между унциями и фунтами — сложнее

  • Преобразование между тоннами и фунтами — проще
  • Преобразование между тоннами и фунтами — сложнее

  • Преобразование между тоннами, фунтами и унциями с десятичными знаками — используйте калькулятор
  • Преобразование между чашками, пинтами и квартами
  • Преобразование между чашками, пинтами, квартами и галлонами
  • Преобразование между унциями, чашками и квартами

  • Все обычные единицы, кроме миль — смешанная практика
  • Все обычные единицы, кроме миль — смешанная практика — задача
  • Преобразование между различными обычными единицами с десятичными знаками — используйте калькулятор

Метрическая система

  • Преобразование между мм, см и м — с использованием десятичных знаков
  • Преобразование между мм, см, м и км — с использованием десятичных знаков
  • Преобразование между мл и л и г и кг — с использованием десятичных знаков

  • Все упомянутые выше метрические единицы — смешанная практика — с использованием десятичных знаков

  • Метрическая система: перевод единиц длины (мм, см, дм, м, плотина, гм, км)
  • Метрическая система: перевод единиц веса (мг, cg, dg, g, dag, hg, kg)
  • Метрическая система: преобразование единиц объема (мл, кл, дл, л, дал, гл, кл)
  • Метрическая система: преобразование единиц длины, веса и объема

Сетка координат

Геометрия


Если вы хотите иметь больший контроль над такими параметрами, как количество проблем, размер шрифта, интервал проблем или диапазон чисел, просто щелкните по этим ссылкам, чтобы самостоятельно использовать генераторы рабочих листов:


Учебный план 5-го класса

Ниже приведены необходимые навыки со ссылками на ресурсы, которые помогут в освоении этого навыка. Мы также поощряем много упражнений и книжную работу. Curriculum Home

Важно: это только руководство.
Обратитесь в местный орган управления образованием, чтобы узнать их требования.

Класс 5 | Умножение

☐ Используйте различные стратегии для умножения трехзначных чисел на трехзначные числа. Примечание. Умножение на что-либо большее, чем трехзначный множитель / множимое, должно выполняться с использованием технологий.

☐ Уметь умножать на отрицательные числа

☐ Развивайте беглость речи с помощью умножения до 12x

5 класс | Раздел

☐ Используйте различные стратегии для деления трех- или четырехзначных чисел на одно- или двузначные числа. Примечание. Деление на что-либо большее, чем двузначный делитель, должно производиться с использованием технологий.

☐ Легко проверить, может ли одно число быть равномерно разделено на другое, используя правила делимости.

5 класс | Числа

☐ Чтение и запись целых чисел в миллионы

☐ Помните, что некоторые числа делятся только на одно и сами по себе (простое число), а другие имеют несколько делителей (составные)

☐ Вычислить кратное целого числа и наименьшее общее кратное двух чисел

☐ Определите факторы данного числа

☐ Найдите общие делители и наибольший общий делитель двух чисел

☐ Вычислить арифметическое выражение, используя порядок операций, включая умножение, деление, сложение, вычитание и скобки

☐ Сравните и закажите номера в миллионы

☐ Округление чисел до сотых и до ближайших 10 000

☐ Поймите структуру разрядных значений десятичной системы счисления: * 10 единиц = 1 десятка * 10 десятков = 1 сотня * 10 сот = 1 тысяча * 10 тысяч = 1 десять тысяч * 10 десятков тысяч = 1 сотня тысяч * 10 сотен тысяч = 1 миллион

☐ Поймите разницу между множителем и кратным целому числу,

5 класс | Десятичные дроби

☐ Сравните десятичные дроби, используя, или =

☐ Используйте различные стратегии для сложения, вычитания, умножения и деления десятичных знаков до тысячных

☐ Чтение, запись и порядок десятичных дробей с точностью до тысячных

☐ Конвертировать проценты в десятичные числа

5 класс | Дроби

☐ Упростите дроби до наименьших значений

☐ Преобразование неправильных дробей в смешанные числа и смешанных чисел в неправильные дроби

☐ Используйте различные стратегии для сложения и вычитания дробей с одинаковыми знаменателями

☐ Сложить и вычесть смешанные числа с одинаковыми знаменателями

☐ Оценивайте суммы и разности дробей с одинаковыми знаменателями.

☐ Создайте эквивалентные дроби, учитывая дробь

☐ Сравните и упорядочьте дроби, включая отличные от знаменателей (с использованием числовой линии и без нее). Примечание: часто используемые дроби, такие как те, которые могут быть указаны на линейке, мерной чашке и т. Д.

☐ Сравните дроби, используя, или =

☐ Сложение, вычитание, умножение и деление дробей (включая смешанные дроби), знаменателями которых являются степени десятичных дробей.

☐ Разделите дробь на целое число

☐ Умножить дробь на целое число

☐ Выражайте десятичные дроби как эквивалент дробей со знаменателями, которые являются степенями 10

☐ Перевести проценты в дроби

5 класс | Проценты

☐ Поймите, что процент означает часть 100

☐ Преобразование дробей или десятичных знаков в проценты

5 класс | Передаточные числа

☐ Понять концепцию передаточного числа

☐ Экспресс-коэффициенты в разных формах

5 класс | Измерение

☐ Используйте линейку для измерения с точностью до сантиметра или миллиметра

☐ Определите личные рекомендации для метрических единиц длины

☐ Определите общепринятые эквивалентные единицы длины (метрические)

☐ Преобразование длины в метрическую систему

☐ Определите инструменты и методы, необходимые для измерения с надлежащим уровнем точности: длины и углы

☐ Измерьте и начертите углы с помощью транспортира

☐ Понять температуру

☐ Понимание температурных шкал Цельсия и Фаренгейта, включая точки замерзания и кипения воды по двум шкалам

☐ Используйте линейку для измерения с точностью до дюйма, 1/2, 1/4 или 1/8 дюйма

☐ Определите личные рекомендации для стандартных единиц длины США

☐ Определите общепринятые эквивалентные единицы длины (США)

☐ Преобразование длины в системе США

5 класс | Время

☐ Вычислить прошедшее время в часах и минутах

☐ Преобразование времени из минут и секунд в секунды или часов и минут в минуты

5 класс | Геометрия (плоскость)

☐ Вычислить формулу периметра для заданных входных значений

☐ Рассчитать периметр правильных и неправильных многоугольников

☐ Определить соответствующие части равных треугольников

☐ Определить и провести линии симметрии основных геометрических фигур

☐ Постройте точки для образования основных геометрических фигур (определите и классифицируйте)

☐ Расчет периметра основных геометрических фигур, нарисованных на координатной плоскости (прямоугольники и фигуры, состоящие из прямоугольников, имеющих стороны с целой длиной и параллельных осям)

☐ Обозначить пары одинаковых треугольников или других геометрических фигур

☐ Определить соотношение сторон одинаковых треугольников

☐ Классифицируйте четырехугольники по свойствам их углов и сторон

☐ Знайте, что сумма внутренних углов четырехугольника составляет 360 градусов

☐ Классифицируйте треугольники по свойствам их углов и сторон

☐ Знайте, что сумма внутренних углов треугольника составляет 180 градусов

☐ Найдите недостающий угол, если даны два угла треугольника

☐ Определить пары совпадающих треугольников или других геометрических фигур

☐ Знайте, что прямой угол равен 90 градусам, прямой угол — 180 градусов, а полный круг — 360 градусов.

☐ Поймите, что подразумевается под порядком симметрии вращения плоской формы, и узнайте, как найти его значение.

☐ Разберитесь, что подразумевается под правильными и неправильными многоугольниками; выпуклые и вогнутые многоугольники; и сложные многоугольники.

☐ Понять, что подразумевается под точечной симметрией

☐ Понять, что подразумевается под вершиной угла и прилегающими углами

5 класс | Геометрия (твердое тело)

☐ Понять платоновы тела

☐ Постройте модели Платоновых тел из их сетей.

☐ Понимать многогранники и классифицировать их как Платоновы тела, призмы, пирамиды и т. Д.

5 класс | Алгебра

☐ Определите и используйте соответствующую терминологию при обращении к константам, переменным и алгебраическим выражениям

☐ Перевести простые словесные выражения в алгебраические

☐ Подставить присвоенные значения в выражения переменных и оценить, используя порядок операций

☐ Решите простые одношаговые уравнения, используя основные целочисленные факты

☐ Решите и объясните простые одношаговые уравнения, используя обратные операции с целыми числами

☐ Создавайте и объясняйте закономерности и алгебраические отношения (пример: 2,4,6,8… алгебраически 2n (удвоение))

☐ Создавайте алгебраические или геометрические узоры, используя конкретные объекты или визуальные рисунки (например, вращайте и закрашивайте геометрические фигуры)

☐ Знайте разницу между закрытым и открытым предложением.

5 класс | Координаты

☐ Определите и нанесите точки в первом квадранте

5 класс | Логика

☐ Используйте логические рассуждения для решения задач, требующих различных навыков

5 класс | Данные

☐ Собирать и записывать данные из различных источников (например,г., газеты, журналы, опросы, диаграммы и обзоры)

☐ Отображение данных в виде линейного графика, чтобы показать увеличение или уменьшение с течением времени

☐ Вычислить среднее значение для данного набора данных и использовать для описания набора данных

☐ Формулируйте выводы и делайте прогнозы по графикам

5 класс | Оценка

☐ Обосновать обоснованность оценки

☐ Оценка сумм, разностей, произведений и частных десятичных знаков

☐ Обоснуйте обоснованность ответов с помощью оценки

5 класс | Вероятность

☐ Перечислите возможные результаты для эксперимента с одним событием

☐ Запишите результаты эксперимента с использованием дробей / соотношений

☐ Создайте образец пространства и определите вероятность одного события, учитывая простой эксперимент (например,г. , катящий числовой куб)

☐ Найдите вероятности в строке числа вероятностей

Важные математические навыки для пятиклассников

Хотите помочь своему пятикласснику освоить математику? Вот некоторые из навыков, которые ваш пятиклассник будет изучать в классе.

Сложение, вычитание, умножение и деление

Многозначные целые числа

Быстро и точно умножайте многозначные целые числа. Разделите целые числа (до четырех цифр) на двузначные числа.

Пример:

Решить 4,824 ÷ 12 =?

Объясните или проиллюстрируйте, как вы решили эту проблему.

Совет: выделите практическое применение математики.

По мере того, как математика, которую они изучают, становится более сложной и менее очевидно связанной с их повседневным опытом, у некоторых детей начинает развиваться математическая тревога. Важно, чтобы ваш ребенок занимался математикой и помогал ему понять, как в реальной жизни применяются концепции, которые ребенок изучает в школе. Составление бюджета на школьные принадлежности или их ежемесячное пособие — один из способов практиковать сложение и вычитание. Если вы попросите их помочь вам с приготовлением или выпечкой, это покажет им, как работают дроби. Помогать рассчитывать цены при покупке продуктов — тоже хорошая практика.

Связанные

Понимание разряда

Расширьте понимание разряда: в многозначном числе цифра в одном месте представляет 1⁄10 того, что она представляет в месте слева от него, и в 10 раз больше как он изображен справа от него.

Сравнение десятичных знаков

Чтение, запись и сравнение десятичных знаков с разрядами тысячных, используя символы> (больше чем) и

  • Прочтите это десятичное число: 23,002.
  • Запишите две и шестьдесят две тысячные в виде десятичного числа.
  • Какой знак подтверждает это утверждение: 5,389 _? _ 5,420
  • Исследователь измеряет количество бактерий, выросших на образцах неохлажденных продуктов. Ваш ребенок насчитывает 73.343 миллиона бактерий в образце A, 73,431 миллиона бактерий в образце B и 74,399 миллиона бактерий в образце C. Расположите образцы в порядке от наибольшего количества бактерий к наименьшему. Объясните или проиллюстрируйте, как вы приводите эти образцы в порядок.
Связанные

Десятичные дроби с точностью до сотых

Сложение, вычитание, умножение и деление десятичных долей с точностью до сотых.

Совет: потренируйтесь в вычислениях с использованием десятичных знаков.

Свяжите работу с десятичными знаками, которую ваш ребенок делает в классе, с реальным миром, поощряя их делать покупки по выгодным ценам.Попросите их разделить стоимость товаров, упакованных оптом, на количество отдельных товаров, чтобы определить стоимость каждого товара. Итак, сколько вы платите за рулон бумажного полотенца или за банку газировки при покупке оптом? Или попросите ребенка подсчитать, сколько вы сэкономите на каждом товаре, если цены со скидкой предполагают оптовые скидки.

Что такое показатель степени

Понять, что такое показатель степени. Например, «2» в 10² указывает, сколько раз нужно умножить число само на себя. 10² можно читать как «10 в степени 2», «10 в степени 2» или «10 в квадрате» и означает 10 x 10 или 100.10³ (или «10 в третьей степени» или «10 в кубе») означает 10 x 10 x 10 или 1000.

Дроби

Решение задач со словами

Решение задач со словами, включающих сложение и вычитание дробей.

Пример:

Пятый класс собирает пазл из 600 деталей. Они начали вчера и собрали 100 частей — всего одну шестую (1⁄6) головоломки. Сегодня их собрано 400 штук. Какая часть головоломки завершена? Нарисуйте картинку И запишите математику, чтобы показать, как вы решили задачу.

Совет: выделите практическое применение математики.

По мере того, как математика, которую они изучают, становится более сложной и менее очевидно связанной с их повседневным опытом, у некоторых детей начинает развиваться математическая тревога. Важно, чтобы ваш ребенок занимался математикой и помогал ему понять, как в реальной жизни применяются концепции, которые он изучает в школе. Составление бюджета на школьные принадлежности или ежемесячное пособие — один из способов для нее практиковать сложение и вычитание.Если вы попросите ее помочь вам с приготовлением или выпечкой, это покажет ей, как работают дроби. Помогать рассчитывать цены при покупке продуктов — тоже хорошая практика.

Нахождение общего знаменателя

Решите задачи со словами, включающие сложение и вычитание дробей с разными знаменателями (нижние числа), преобразовывая их в дроби с одинаковым знаменателем, называемые общим знаменателем.

Пример:

Самая высокая девочка в пятом классе имеет рост 51 7⁄8 дюйма.Самый высокий мальчик в пятом классе имеет рост 49 1⁄2 дюйма. Какая разница в их росте?

После вечеринки остались две чашки лимонада. В одной миске 1⁄3 галлона. В другом — 1⁄2 галлона лимонада. Друг говорит, что не стоит пытаться объединить их в 1-галлонный контейнер, потому что лимонад вытечет наверх. Ты согласен? Почему или почему нет?

Умножение дробей

Решайте задачи со словами, включающие умножение дробей на другие дроби и умножение дробей на смешанные числа (целое число и дробь, например 11⁄4 или 21⁄2).

Пример:

  • В оркестре средней школы 1⁄3 учащихся-музыкантов играют на струнных инструментах. Из учеников, играющих на струнных инструментах, 3⁄4 играют на скрипке. Какая часть оркестра играет на скрипке?
  • Утром во время экскурсии в яблоневый сад пятиклассники собрали 4⁄5 бушеля яблок. После обеда в полдень они собрали в 2,5 раза больше яблок. Уместятся ли все яблоки, собранные ими днем, в ящик на 2 бушеля? Откуда вы знаете?

Совет: потренируйтесь использовать дроби.

Помогите своему ребенку познакомиться с дробями, попросив его масштабировать рецепты для вашей семьи. Пусть они начнут с того, что уменьшат рецепт вдвое или вдвое. Когда они почувствуют себя комфортно, попросите их преобразовать его на 1 1/2, чтобы рецепт, который должен был накормить семью из четырех человек, работал на семью из шести человек.

Единица деления дробей

Разделите единичные дроби (дроби с 1 в числителе или верхним числом) на целые числа. Разделите целые числа на единичные дроби.

Пример:

Если три человека разделят ½ фунта шоколада поровну, сколько шоколада получит каждый? Объясните или проиллюстрируйте, как вы решили эту проблему.

Умножение на дроби

Помните, что умножение числа на дробь меньше 1 даст ответ меньше числа — например: 12 x ¾ = 9. Умножение числа на дробь больше 1 даст результат в ответе больше числа — например: 12 x 2 ½ = 30.

Измерения и данные

Преобразование единиц и дробей

Преобразование единиц и долей единиц в одной системе измерения.

Пример:

Сколько минут составляет 1⁄5 часа? Объясните или проиллюстрируйте, как вы решили эту проблему.

Проблемы многоступенчатого преобразования единиц измерения

Решайте многоступенчатые задачи преобразования слов, используя преобразование стандартных единиц измерения разного размера.

Пример:

У меня 75 см ленты.Для выполнения проекта мне нужно в семь раз больше ленты. Сколько еще метров ленты мне нужно?

Объясните или проиллюстрируйте, как вы решили эту проблему.

Использование линейного графика

Решайте проблемы, используя информацию (в единицах дроби), представленную на линейном графике.

Геометрия

Понимание объема

Под объемом понимается измерение пространства внутри трехмерной или твердой фигуры. Используйте формулы длина x ширина x высота или основание x высота , чтобы измерить объем трехмерного или твердого объекта с прямоугольными сторонами, например куба. Измеряйте объем для решения реальных проблем.

Пример:

Прямоугольный контейнер для мороженого имеет длину 8 дюймов и высоту 4 дюйма. Каков объем контейнера, выраженный в кубических дюймах?

Советы, которые помогут вашему пятикласснику в уроке математики, можно найти на нашей странице с советами по математике для пятого класса.

Ресурсы Parent Toolkit были разработаны NBC News Learn с помощью профильных экспертов и соответствуют Общим основным государственным стандартам.

Математика Мамонт 5 класс Полная программа

Math Mammoth Grade 5 Grade Complete Curriculum — это полная математическая программа для 5-го класса, в которой вы найдете все необходимое для обучения математике в 5-м классе. Он доступен как в версии для скачивания, так и в виде печатных копий.

208 страниц
170 страниц уроков


201 страниц
176 страниц уроков

Содержимое 5-A и образцы
Содержимое и образцы 5-B
Руководство пользователя, класс 5
Выравнивание / содержимое CCS
FAQ


Цены и заказ

Вы можете приобрести Math Mammoth Grade 5 для загрузки, на компакт-диске или в виде уже распечатанных книг.


Цифровая версия

5 класс: $ 39,50 (скачать)

(два рабочих текста, ключи ответов, тесты, совокупные обзоры, средство создания рабочих листов, Soft-Pak)

Только часть 5-A:
$ 19,75 (загрузить)

(Все для первой половины 5-го класса по математике; включает Софт-Пак)

Только часть 5-B:
$ 19,75 (загрузить)

(Все для второй половины 5-го класса по математике; включает Софт-Пак)

Вы будете покупать загружаемые материалы у моего авторизованного реселлера Comecero, LLC.


CD

Полная оценка 5: $ 44,50

(два рабочих текста, ключи ответов, тесты, совокупные обзоры, средство создания рабочих листов, Soft-Pak)



Печатные копии

Ресурсный центр Rainbow продает тексты с идеальным переплетом либо с полноцветными, либо с внутренними страницами в оттенках серого:

Математика Мамонт 5 класс (ч / б)
Математика Мамонт 5 класс (цвет)

Lulu продает в спиральном переплете (катушку) версии учебников для студентов и книг с тестами и обзорами.

Рабочий текст 5-A $ 18,45
Рабочий текст 5-B $ 18,45
Тесты и совокупные обзоры 10,25 $
Ключи ответов $ 12,95 (идеальная привязка)


Вспомогательные материалы

Включает ключи ответов, тесты, совокупные обзоры, средство создания рабочих листов и программное обеспечение Soft-Pak (цифровая загрузка). Купите это вместе с печатными учебниками для студентов. Не покупайте, если вы получили полную цифровую версию, как указано выше.

, издание 2019 г. (8,95 долл. США):

издание 2020 г. (8 долларов США.95):



План урока

Вы можете приобрести план уроков по математике Mammoth Grade 5, чтобы использовать его в онлайн-планировщике Homeschool Planet . Узнать больше.

В основе учебной программы лежат два студенческих рабочих текста (A и B), каждый из которых охватывает примерно полгода работы по математике. Эти рабочие тексты содержат все инструкции и упражнения из одной книги, что упрощает подготовку к уроку. Рабочие тексты написаны непосредственно ученику, и, таким образом, они позволяют многим ученикам учиться самостоятельно и учиться прямо из книг.

Вы также получите отдельные ключи ответов, тесты по главам, дополнительные совокупные обзоры (я рекомендую вам использовать хотя бы некоторые из них) и универсальное средство для создания рабочих листов (требуется доступ в Интернет) для тех случаев, когда вашему ребенку нужно немного больше практики .


Характеристики

  • Math Mammoth фокусируется на концептуальном понимании . В нем объясняется «ПОЧЕМУ», так что ваши дети могут понимать математику, а не просто узнавать «КАК» это делать.
  • Концепции часто объясняются с помощью визуальных моделей , за которыми следуют упражнения с использованием этих моделей.Эти визуальные модели могут заменить многие дети манипулятивными средствами; однако при желании очень легко добавить к урокам соответствующие манипуляторы.
  • Учебная программа ориентирована на овладение знаниями . Это означает, что он довольно долго концентрируется на теме, вникая в ее различные аспекты. Это способствует концептуальному пониманию, в отличие от спиральных учебных программ, которые часто имеют тенденцию слишком много прыгать от темы к теме.
  • Особое внимание уделяется умственной математике и пониманию чисел .
  • Требуется очень небольшая подготовка учителей .
  • Учебная программа не имеет отдельного руководства для учителя и не написана по сценарию. Во введении к каждой главе есть некоторые примечания для учителя относительно материала главы. Все инструкции написаны непосредственно для ученика в рабочем тексте, а также существуют сопроводительные видеоролики, где вы можете увидеть, как Мария сама преподает материал.
  • После введения каждой главы вы найдете список интернет-ссылок и ресурсов (игры, викторины, анимации и т. Д.).), которые можно использовать для развлечения, иллюстраций и дальнейшей практики.

Дополнительные возможности для цифровой версии (скачать / CD)

Файлы PDF включены для аннотации. Это означает, что при желании ваш ученик может заполнить их на компьютере, используя пишущую машинку и инструменты рисования в Acrobat Reader версии 9 или выше, или на планшетном устройстве, используя любое приложение PDF с возможностью аннотации. (Узнать больше.)

БОНУС! Если вы приобретете загружаемую версию или версию на компакт-диске, вы также получите программы Soft-Pak полностью БЕСПЛАТНО . 6 программ Soft-Pak (4 математических, 1 языковая, 1 составитель списков) предлагают как экранные, так и печатные задания в формате с низким содержанием графики и высоким содержанием. Подробнее читайте и смотрите скриншоты.


Обзор тем

Основными направлениями обучения в Math Mammoth Grade 5 являются:

  • четыре операции с целыми числами
  • разряда с большими числами и разумное использование калькулятора
  • решение задач и простые уравнения
  • десятичная арифметика (все операции с десятичными знаками)
  • статистика и графики
  • дробная арифметика (все операции с дробями)
  • геометрия: обзор углов, площади и периметра; рисование кругов; классификация треугольников и четырехугольников; объем прямоугольных призм

См. Также оглавление 5-A и 5-B (в файлах примеров), которое позволит вам увидеть затронутые темы более подробно.


Дополнения

План урока

Мы предлагаем план уроков по математике Mammoth Grade 5 для онлайн-планировщика Homeschool Planet . Он доступен в двух версиях: обычный и план PLUS, который также включает файл плана в формате PDF. Узнать больше.


Рабочая тетрадь по обзору навыков

Рабочая тетрадь по обзору навыков мамонта по математике, 5 класс предоставляет дополнительную практику по темам учебной программы по математике мамонта для 5 класса. Я рекомендую вам приобретать его только в том случае, если вы уверены, что вашему ученику действительно нужна дополнительная практика.Узнать больше.


Введение

Пятый сорт — это время для дробей и десятичных знаков, в частности. Мы изучаем дроби и десятичные дроби, а также операции с ними глубоко и детально. Студенты также углубляют свое понимание целых чисел, узнают больше о решении задач и знакомятся с калькулятором.

Год начинается с изучения целых чисел и их операций. Студенты могут просмотреть умножение нескольких цифр и выучить деление в столбик с двузначными делителями .Мы также рассматриваем делимость и множители из четвертого класса и изучаем разложение на простые множители .

Во второй главе основное внимание уделяется большим числам и с помощью калькулятора . Это первый раз, когда калькулятор вводится в полную учебную программу Math Mammoth — до сих пор все вычисления производились мысленно или с помощью бумаги и карандаша. Я хочу, чтобы учащиеся научились критически относиться к калькулятору и использовали его с здравым смыслом.Каждое упражнение, в котором разрешено использование калькулятора, помечено маленьким символом калькулятора.

Третья глава посвящена уравнениям и решению задач . Изучаем простые уравнения с помощью весов и стержней модели . Основная идея состоит в том, чтобы научить студентов понимать уравнение и то, что это значит для решения уравнения. Студенты также решают изрядное количество задач, используя модель визуальной панели.

В четвертой главе примерно знаков после запятой и некоторые операции с десятичными знаками (остальные будут изучены в главе 6).Пятый класс — это время, когда ученики изучают все основные операции с десятичными знаками. В этом В главе мы сосредоточимся на разрядах с десятичными знаками, сложении и вычитании десятичных знаков, а также умножение и деление десятичных знаков на целые числа. Умножение десятичных знаков на десятичные и разделение десятичных знаков на десятичные дроби рассматривается в главе 6.

В главе 5 мы изучаем график в координатной сетке, линейные и гистограммы, а также среднее значение и режим . Сегодняшний мир становится все более сложным из-за большого количества данных, представленных в СМИ, поэтому наши детям нужно хорошо разбираться в статистических графиках, чтобы понимать всю эту информацию.

Глава 6 продолжает наше исследование десятичных знаков . Основное внимание уделяется умножению десятичных знаков на десятичные дроби, делению десятичные дроби на десятичные дроби и преобразование между единицами измерения.

Глава 7 охватывает сложение и вычитание дробей — еще одна тема для 5-го класса, помимо десятичных дробей. Самая сложная тема этой главы — сложение и вычитание непохожих дробей. выполняется путем их предварительного преобразования в эквивалентные дроби с общим знаменателем.

В главе 8 мы изучаем умножение и деление дробей под разными углами.

Глава 9 переносит нас к геометрии, начиная с обзора углов и многоугольников. Оттуда студенты будут научиться рисовать круги, классифицировать треугольники и четырехугольники и концепцию тома в контексте правильные прямоугольные призмы (коробки).


Манипуляторы

Вот список манипуляторов, которые необходимы или рекомендуются для 5 класса.

  1. Линейка в дюймах (для 5-B).
  2. Линейка сантиметров (для 5-B).
  3. Циркуль и транспортир (для 5-Б).
  4. Фракционные манипуляторы. Учебная программа содержит печатные формы для создания собственных манипуляторов с дробями. В тексте широко используются визуальные модели круговых диаграмм, и студенты даже учатся рисовать их сами для упражнений, поэтому изготовление или покупка манипуляторов является совершенно необязательным (необязательно; для 5-B).

Дополнения

План урока

За 9 долларов.95, вы можете приобрести план уроков по математике Mammoth Grade 3, чтобы использовать его в онлайн-планировщике Homeschool Planet . Узнать больше.


Обзоры и характеристики

Math Mammoth обзор на As They Grow Up блог

Обзор использования Math Mammoth 5-го класса с аутичным сыном в Через открытое окно блог

Это наш первый год обучения на дому, потому что мы хотели путешествовать. Мы начали выпускать Singapore Standard Edition, потому что этим она занималась в своей бывшей школе.Я сразу понимаю, что, несмотря на степень магистра делового администрирования и неплохое математическое образование, я понятия не имел, что мне делать, чтобы преподавать уроки. Итак, мы заявили, что ищем что-то еще, сложное, но что я могу понять и чему научить. Мы выбрали MM, а также используем CTC Math из-за видео, идеального сочетания. Мне нравится, что MM можно распечатать, потому что у нас не всегда есть подключение к Интернету в дороге, и они выставляют нам все оценки. Поскольку Сингапур пропускает некоторые основы в младших классах, я совмещаю уроки MM в 4-м и 5-м классах, чтобы определить, что нужно моему ребенку.Отличная программа, много практики и примеров, в отличие от Сингапура, действительно проясняют цель урока. Я очень рекомендую.

MaryLu C
Октябрь 2016

Моим детям (первоклассникам и пятиклассникам) очень понравился Math Mammoth. Мне нравится, что я печатаю только то, что мне нужно, и им нравится, чтобы их не забрасывали слишком большим количеством проблем на концепцию. Как только они его получат, мы двинемся дальше. Мне нравятся такие занятия, как «Деньги», которые я могу использовать с обоими детьми. Очень рекомендую Math Mammoth!

— Кэрри В., Член кооператива
мая 2015 г.

Обзор

: Math Mammoth (Revisited) — Введение в дроби, раздел 2, светло-голубой класс 5 от Got Chai? блог.

Большое спасибо, Мария. Мы вас обожаем! Я действительно ценю доступ к такой исключительной программе. Что Бог послал. Вы являетесь таким благословением для нашей семьи и нашего сообщества, обучающегося на дому. Это лучшая математическая программа, которую мы когда-либо видели. И мы перепробовали многие за эти годы … Саксонию, Сингапур, Калверт и одну в системе государственных школ, которую мы смогли взять взаймы.Это бьет их руки вниз. Это основательно, хорошо продумано и заставляет наших детей думать о математике. Практики и повторений достаточно, чтобы концепции действительно усвоились, не утомляя их. Это единственная математическая программа, которую мы использовали, и мне не пришлось запускать ее в поисках добавки. Еще раз спасибо, Мария, очень рада, что мы тебя нашли.

Дженнифер
августа 2014

Примечание: два обзора ниже относятся к более старой версии MM5. Однако принципиально он похож на новую, исправленную версию.

Зейн отлично учится по математике! Мне эта программа нравится намного больше, чем Saxon … мы уделяем достаточно времени каждой новой концепции, чтобы он действительно ее «понял»! Скачанный материал (5 класс) намного красочнее (и поэтому вызывает у него интерес!), Чем печатная версия Math Mammoth [использованная в 4 классе]. И со всей дополнительной «помощью», которую вы предоставляете; от подсказок, предложений и игр до списка других сайтов с играми, загрузками (фаворитом был math rider!) и халявой… вы сделали математику УДОВОЛЬСТВИЕМ для ОБЕИХ из нас! Я действительно не могу вас отблагодарить (а я уже обучил на дому двух мальчиков . .. один разработчик компьютерных веб-сайтов, а другой инженер-химик … так что я СТАРАЯ мама и, честно говоря, готова немного повеселиться с учеба в школе)!

Math Mammoth УДИВИТЕЛЬНЫЙ!

Бонни
января 2012

Я мать двух прекрасных дочерей 13 и 11 лет, которые учатся в государственной школе французского погружения на острове Ванкувер, Канада.

У моей младшей дочери был трудный 5-й класс по математике, и в школе ей оказывали дополнительную помощь в обучении.Я часто беспокоился о том, что, поскольку все обучение ведется на французском, ее втором языке, ей в конечном итоге может быть труднее понять уроки математики или, возможно, способ ее преподавания. И хотя я пытался помочь ей в течение учебного года, это стало скорее упражнением в переводе, потому что я только год изучал базовый вводный французский в университете, и ей пришлось перевести мне вопрос, прежде чем я смог помочь.

Я знал, что нам нужно поработать над этим летом.Я узнал, что в ее математических знаниях было больше пробелов, чем я думал ранее. Именно тогда я отказался от обучения ее математике на французском языке и начал искать рабочие листы в Интернете. Я был очень рад встретить ваши бесплатные рабочие листы, но после просмотра полных страниц с образцами рабочего текста Mammoth и клипов на Youtube, которые помогают обучать «учителя», я был так впечатлен, что купил голубой рабочий текст A и B. Мне очень нравится, насколько ясно объясняются уроки и как концепции плавно сочетаются друг с другом, и что вы персонализировали этот курс с инструкциями для «учителей».И когда я сказал своей дочери, что нашел в Интернете учителя математики, который может научить МЕНЯ преподавать, она, похоже, была довольна и, возможно, немного вздохнула (!!?)

Итак, вот и мы, день 4, страница 17, она набирает скорость, и ее отношение улучшилось, поскольку она начинает понимать уже пройденную математику. Мне очень нравится, что уроки или правила выделены или обрисованы в общих чертах, чтобы к ним было легче вернуться. Рабочий лист также выглядит привлекательно. Я надеюсь, что этим летом мы сможем пройти весь курс, не будучи слишком утомительным, но я вижу, что она набирает чувство собственного достоинства и воодушевляется, она работает, задавая вопросы, с меньшим руководством с моей стороны.

Итак, большое спасибо, Мария!

Я ожидаю, что куплю голубой рабочий текст 6-го класса, чтобы «работать вместе» с ней, когда она вернется к математике в 6-м классе по французскому языку в сентябре. Я уверен, что смогу отслеживать ее успехи, а затем предлагать помощь по мере необходимости (или, если необходимо!).

С уважением, Элейн 900 — 13 июля 2010 г.


% PDF-1.4 % 448 0 объект > эндобдж xref 448 82 0000000016 00000 н. 0000002775 00000 н. 0000002922 00000 н. 0000003419 00000 п. 0000003751 00000 п. 0000004062 00000 н. 0000004260 00000 н. 0000004374 00000 п. 0000004486 00000 н. 0000004599 00000 н. 0000004708 00000 н. 0000005172 00000 п. 0000005199 00000 п. 0000005635 00000 п. 0000005662 00000 н. 0000006228 00000 п. 0000006624 00000 н. 0000006651 00000 п. 0000006678 00000 н. 0000007142 00000 п. 0000007282 00000 н. 0000007415 00000 н. 0000007550 00000 н. 0000007687 00000 н. 0000008233 00000 н. 0000008771 00000 н. 0000009330 00000 н. 0000009891 00000 н. 0000010435 00000 п. 0000010462 00000 п. 0000010856 00000 п. 0000010994 00000 п. 0000011520 00000 п. 0000011783 00000 п. 0000012286 00000 п. 0000012575 00000 п. 0000012846 00000 п. 0000014112 00000 п. 0000025731 00000 п. 0000025844 00000 п. 0000027008 00000 н. 0000027316 00000 н. 0000027431 00000 н. 0000030282 00000 п. 0000030600 00000 п. 0000030719 00000 п. 0000031960 00000 п. 0000032273 00000 п. 0000032400 00000 п. 0000034380 00000 п. 0000034704 00000 п. 0000034774 00000 п. 0000034854 00000 п. 0000038726 00000 п. 0000039000 00000 н. 0000039288 00000 п. 0000039358 00000 п. 0000039438 00000 п. 0000044340 00000 п. 0000044609 00000 п.

методика выполнения операций и примеры реализации алгоритма

Математика

12.11.21

9 мин.

Одними из базовых математических операций являются умножение и деление натуральных чисел. В 5 классе они изучаются более подробно с учетом всех «тонкостей». Специалисты разработали специальные алгоритмы, позволяющие произвести расчеты без ошибок. Однако перед обучением нужно разобрать основные определения и понятия, а затем приступать к практике.

Оглавление:

  • Общие сведения
  • Методика умножения
  • Алгоритм деления
  • Примеры решений

Общие сведения

Умножением двух или нескольких чисел называется математическая операция, состоящая из множителей и результата (произведения). Следует отметить, что ее можно заменить сложением, т. е. 2*3=2+2+2=6. Иными словами, первый множитель указывает на число, а второй обозначает, какое количество раз его необходимо сложить с эквивалентным значением. Операция умножения (при не табличных значениях) выполняется в столбик.

Деление — вид арифметической операции, при которой одно число (делимое) делится на другое (делитель), а их результатом является частное. Ее можно заменить вычитанием, т. е. 36/6=36−6−6−6−6−6−6=0. Следует отметить, что деление является сложным типом операции, поскольку классифицируется на два вида: без остатка и с его наличием.

Натуральные — числа, при помощи которых производится счет, т. е. 1, 2, 3, 4 и т. д. Весь числовой ряд является натуральным. Он расположен только в порядке возрастания.

Методика умножения

При умножении чисел используется специальная методика, при которой два значения записываются в столбик: единицы под единицами, десятки под десятками и т. д. Алгоритм возможно применять для многозначных величин. Он имеет следующий вид:

  1. Написать числовые пары одну под другой.
  2. Выполнить умножение единиц второго числа на первое.
  3. Записать результат.
  4. Перемножить разряд десятков с первым значением и записать его со смещением влево.
  5. Повторить четвертый пункт методики (нужно не забывать о постоянном смещении влево на один разряд).
  6. Сложить обе величины и записать окончательный результат.

Следует отметить, что для оптимизации решения примеров по математике 5 класса на умножение применяются свойства. К ним относятся следующие:

  1. При произведении любого натурального числа на 0 получается нулевое значение.
  2. Произведение двух чисел, одно из которых 1, есть другое значение, т. е. а*1=а.
  3. Переместительное: перемена мест сомножителей не меняет результат (mnо=mоn=nоm=nmo=omn=onm).
  4. Сочетательный закон: перемножать можно в любом порядке. Например, если удобно умножить 1 на 3, то можно их перемножить, а затем подключить второе значение.
  5. Распределительное: умножение числа на сумму двух величин эквивалентно произведению общего множителя за скобкой на первое и второе значения, а затем полученные два результата требуется сложить. Это свойство может быть полезно при решении уравнений.

Следует отметить, что любое из вышеописанных свойств возможно применять несколько раз. Например, можно использовать переместительное и сочетательное.

Алгоритм деления

Деление, как и умножение, выполняется по определенному алгоритму. Однако следует учитывать факт, что результат может быть целым и с остатком. В первом и во втором случае применяется аналогичный алгоритм, но существуют некоторые особенности. Методика деления натурального числа на другое значение, принадлежащего к этому типу, имеет такой вид:

  1. Записать делимое, а с правой стороны — делитель, отделяя их прямой чертой.
  2. Взять количество разрядов, соответствующих делителю.
  3. Подобрать множитель, а затем записать его произведение на делитель под I разрядной группой и вычислить их разность.
  4. Взять II разрядную группу и выполнить аналогичную операцию.
  5. Продолжать до того момента, пока не получится 0 или величина (остаток), которая меньше делителя.

Следует отметить, что операция деления имеет несколько особенностей. К ним относятся следующие:

  1. Деление на 0 невозможно. Возможно только нуль разделить на любое число. Результат при этом будет равен 0. Пример деления для 5 класса имеет такой вид: 0/8=0.
  2. Деление на 1 имеет эквивалентное исходное значение: 5/1=5.

После рассмотрения алгоритмов нужно перейти решению примеров на умножение и деление для 5 класса.

Примеры решений

Для примера следует разобрать задачу на перемножение чисел 25 и 25. В этом случае рекомендуется воспользоваться таким алгоритмом:

  1. Записать первый множитель, а под ним второй.
  2. Провести результирующую горизонтальную линию после величин в первом пункте.
  3. Поставить слева знак произведения.
  4. Умножить 25 на 5 (разряд единиц), а затем записать под чертой: 125.
  5. Перемножить 25 и 2 (десятки): 50.
  6. Записать величину, полученную в пятом пункте под десятками первого результата.
  7. Сложить величины 125+50 (смещенное влево): 625.

Следующий пример — задание на операцию деления 625 на 5. Для этого следует воспользоваться вышеописанной методикой:

  1. Написать делимое и делитель, разделив их вертикальной чертой.
  2. Рассмотреть I разряд. Он делится на 5, но с остатком: 6/5=1 (1 — остаток).
  3. Записать в поле результата число 1.
  4. Перемножить 1 и 5, записав под шестеркой: 5.
  5. Отделить остаток: 1.
  6. Перенести к 1 второй разряд: 12.
  7. Подобрать множитель: 5*2=10<12.
  8. Перемножить 5 и 2, записав результат под 12.
  9. Выделить остаток: 2.
  10. Перенести к 2 III разряд: 25.
  11. Осуществить деление 25 на пятерку: 25/5=5.
  12. Искомое частное: 125.

Следует отметить, что множитель при выполнении операции деления подбирается по такому принципу: его произведение на делитель не должно превышать значения делимого. Операция с остатком реализуется аналогично. Например, в выражении 4/3 остаток равен единице, а результат записывается следующим образом: 4/3=1 (+1). Последнее значение «(+1)» указывает на положительный остаток, т. е. 3*1+1=4.

Иногда в математической литературе можно встретить запись «(-1)», которая указывает на отрицательную величину остатка. Например, запись «11 (-1)» при делении неизвестного числа на тройку определяет исходную величину следующим образом: 3*11−1=33−1=32. Пример возможно править и в таком виде: 32/3=10 (+2)=11 (-1).

Таким образом, арифметические операции умножения и деления рекомендуется при отсутствии калькулятора выполнять в столбик, но для этого следует знать основные методики.

Не успеваете написать работу?

Заполните форму и узнайте стоимость

Вид работыПоиск информацииДипломнаяВКРМагистерскаяРефератОтчет по практикеВопросыКурсовая теорияКурсовая практикаДругоеКонтрольная работаРезюмеБизнес-планДиплом MBAЭссеЗащитная речьДиссертацияТестыЗадачиДиплом техническийПлан к дипломуКонцепция к дипломуПакет для защитыСтатьиЧасть дипломаМагистерская диссертацияКандидатская диссертация

Контактные данные — строго конфиденциальны!

Указывайте телефон без ошибок! — потребуется для входа в личный кабинет.

* Нажимая на кнопку, вы даёте согласие на обработку персональных данных и соглашаетесь с политикой конфиденциальности

Подтверждение

Ваша заявка принята.

Ей присвоен номер 0000.
Просьба при ответах не изменять тему письма и присвоенный заявке номер.
В ближайшее время мы свяжемся с Вами.

Ошибка оформления заказа

Кажется вы неправильно указали свой EMAIL, без которого мы не сможем ответить вам.
Пожалуйста проверте заполнение формы и при необходимости скорректируйте данные.

Основные правила математики с примерами. 5 класс

Основные правила математики с примерами. 5 класс

Содержание
  • Натуральные числа
  • Сравнение натуральных чисел
  • Свойства сложения
  • Формула пути
  • Корень уравнения
  • Правила решения уравнений
  • Отрезок, прямая, луч
  • Угол, биссектриса угла
  • Углы: развернутый, прямой, острый, тупой
  • Многоугольники. Равные фигуры
  • Треугольники: остроугольный, прямоугольный, тупоугольный
  • Треугольники: равнобедренный, равносторонний, разносторонний
  • Прямоугольник. Квадрат. Периметр
  • Умножение. Свойства умножения
  • Деление. Деление с остатком
  • Площадь. Площадь квадрата, прямоугольника
  • Объем. Объем прямоугольного параллелепипеда, куба
  • Дроби: правильная, неправильная, сравнение дробей
  • Сложение и вычитание дробей с одинаковыми знаменателями
  • Сложение и вычитание смешанных чисел
  • Преобразование неправильной дроби в смешанное число
  • Преобразование смешанного числа в неправильную дробь
  • Десятичные дроби: свойства, сравнение, округление
  • Десятичные дроби: сложение, вычитание
  • Десятичные дроби: умножение, деление
  • Среднее арифметическое
  • Процент
Натуральные числа

Числа 1, 2, 3, 4, 5, 6, 7, 8, 9, 10, 11, 12 и т. д., которые используют при счете предметов, называют натуральными.

Сравнение натуральных чисел

Число 0 меньше любого натурального числа.

0<1, 0<100

Из двух натуральных чисел, которые имеют разное количество цифр большим является то, у которого количество цифр больше.

4352⏟4>999⏟3

Из двух натуральных чисел с одинаковым количеством цифр большим является то, у которого больше первая (при чтении слева направо) из неодинаковых цифр

3561>3559

Свойства сложения

Переместительный закон: 

15+10=10+15

Сочетательный закон:

(23+15)+25=23+(15+25)

Формула пути

S=V·t,где S — пройденный путь, V — скорость движения, t — время, за которое пройден путь S

 

= 50км,  = 2ч,  = 25км/ч

,   50км = 25км/ч· 2ч

,   25км/ч = 50км : 2ч

,   2ч = 50км : 25км/ч

Корень уравнения

Корнем (решением) уравнения называют число, которое при подстановке его вместо буквы превращает уравнение в верное числовое равенство.

2·x+10=16

x = 3 — корень, так как 2·3+10=16

Что значит «Решить уравнение»

Решить уравнение — это значит найти все его корни или убедиться, что их вообще нет.

Правила решения уравнений
  • Чтобы найти неизвестное слагаемое, надо из суммы вычесть известное слагаемое.

20слагаемое+xслагаемое=100суммаx = 100 — 20x = 80

  • Чтобы найти неизвестное уменьшаемое, надо к разности при­бавить вычитаемое.

xуменьшаемое—10вычитаемое=40разностьx = 40 + 10x = 50

  • Чтобы найти неизвестное вычитаемое, надо из уменьшаемого вычесть разность.

50уменьшаемое—xвычитаемое=40разностьx = 50 — 40x = 10

  • Чтобы найти неизвестный множитель, надо произведение раз­делить на известный множитель.

xмножитель·7множитель=56произведениеx = 56 : 7x = 8

  • Чтобы найти неизвестное делимое, надо делитель умножить на частное.

xделимое:8делитель=9частноеx = 9 · 8x = 72

  • Чтобы найти неизвестный делитель, надо делимое разделить на частное.

42делимое:xделитель=7частноеx = 42 : 7x = 6

Отрезок, прямая, луч
Отрезок

Отрезок — часть прямой, ограниченная двумя точками(концами) и все точки между этими концами(внутренние точки отрезка)

Свойство длины отрезка

Если на отрезке отметить точку , то длина отрезка равна сумме длин отрезков и .

Равные отрезки

Два отрезка называют равными, если они совмещаются при наложении.

Свойство прямой

Через две точки проходит только одна прямая.

Измерить отрезок

Измерить отрезок означает подсчитать, сколько единичных отрезков в нем помещается

Ломаная

Ломаная — геометрическая фигура, состоящая из отрезков, последовательно соединенных друг с другом

Луч

Луч (полупрямая) — это геометрическая фигура, часть прямой, состоящая из точки(начала луча) и всех точек прямой, лежащих по одну сторону от начала луча.В названии луча присутствуют две буквы, например, . Причем первая буква всегда обозначает точку начала луча, поэтому менять местами буквы нельзя.

 

Угол, биссектриса угла
Угол

Фигуру, образованную двумя лучами, имеющими общее начало, называют углом.

Равные углы

Два угла называют равными, если они совмещаются при наложении.

Свойство величины угла

Если между сторонами угла ∠ провести луч , то градусная мера  ∠ равна сумме градусных мер углов ∠ и ∠, то есть ∠ = ∠+ ∠.

Биссектриса угла

Луч, который делит угол на два равных угла, называется биссектрисой угла.

Углы: развернутый, прямой, острый, тупой
Развернутый угол

Угол, стороны которого образуют прямую, называют развернутым. Градусная мера развернутого угла равна 180°.

Прямой угол

Угол, градусная мера которого равна 90°, называют прямым.

Острый угол

Угол, градусная мера которого меньше 90°, называют острым.

Тупой угол

Угол, градусная мера которого больше 90°, но меньше 180°, называют тупым.

 

Многоугольники.
Равные фигуры
Равные многоугольники

Два многоугольники называют равными, если они совмещаются при наложении.

Равные фигуры

Две фигуры называют равными, если они совмещаются при наложении.

Треугольники: остроугольный, прямоугольный, тупоугольный
Остроугольный треугольник

Если все углы треугольника острые, то его называют остроугольным треугольником.

Прямоугольный треугольник

Если один из углов треугольника прямой, то его называют прямоугольным треугольником.

Тупоугольный треугольник

Если один из углов треугольника тупой, то его называют тупоугольным треугольником.

Треугольники: равнобедренный, равносторонний, разносторонний
Равнобедренный треугольник

Если две стороны треугольника равны, то его называют равнобедренным треугольником.

Равносторонний треугольник

Если три стороны треугольника равны, то его называют равносторонним треугольником.

Периметр равностороннего треугольника

Если сторона равностороннего треугольника равна , то его периметр вычисляют по формуле

Разносторонний треугольник

Если три стороны треугольника имеют разную длину, то его называют разносторонним треугольником.

Прямоугольник. Квадрат. Периметр
Прямоугольник

Если в четырехугольнике все углы прямые, то его называют прямоугольником.

Свойство прямоугольника

Противоположные стороны прямоугольника равны.

Периметр прямоугольника

Если соседние стороны прямоугольника равны и , то его периметр вычисляют по формуле

Квадрат

Прямоугольник, у которого все стороны равны, называют квадратом.

Периметр квадрата

Если сторона квадрата равна , то его периметр вычисляют по формуле .

Умножение. Свойства умножения
Умножение
  • Произведением числа на натуральное число , которое не равно 1, называют сумму, состоящую из  слагаемых, каждый из которых равен . В равенства    числа  и называют множителями,  а число и запись  — произведением.

 


  • Если один из двух множителей равен 1, то произведение равно второму множителю.
  • Если один из множителей равен нулю, то произведение равно нулю.
  • Если произведение равно нулю, то хотя бы один из множителей равен нулю.
Свойства умножения
  • Переместительный закон умножения:
  • Сочетательный закон умножения: 
  • Распределительное свойство умножения относительно сложения:  

2·(3+10) = 2·3 + 2·103·11 + 3·4 = 3·(11 + 4)

  • Распределительное свойство умножения относительно вычитания:

2·(15—7) = 2·15 — 2·73·10 — 3·4 = 3·(10 — 4)

Деление. Деление с остатком
Деление

Для натуральных чисел равенство   является правильным, если является правильным равенство

15 : 5 = 3 -правильное равенство, так как  равенство 5 · 3 = 15 верное

В равенстве    число называют делимым, число — делителем, число и   запись  — частным от деления, отношением, долей.

На ноль делить нельзя.

Для любого натурального числа  правильными являются равенства:

,

Деление с остатком

, где  — делимое, — делитель, — неполное частное, — остаток, .

154делимое=50делитель · 3неполное частное + 4остаток,    4<50

Если остаток равен нулю, то говорят, что число делится нацело на число .

Площадь. Площадь квадрата, прямоугольника
Свойства площади фигуры

Равные фигуры имеют равные площади;

Площадь фигуры равна сумме площадей фигур, из которых она состоит.

Площадь прямоугольника

Площадь прямоугольника равна произведению длин его соседних сторон, выраженных в одних и тех же единицах.

Площадь квадрата

,

где  — площадь квадрата,  — длина его стороны.

Объем. Объем прямоугольного параллелепипеда, куба
Свойства объема фигуры

Равные фигуры имеют равные объемы;
Объем фигуры равен сумме объемов фигур, из которых она состоит.

Объем прямоугольного параллелепипеда
  • ,

где — объем параллелепипеда, , и  — его измерения, выраженные в одних и тех же единицах;

, где — площадь поверхности прямоугольного параллелепипеда.

  • ,

где  — площадь основания параллелепипеда, — его высота.

Объем куба

,

где  — объем куба,  — длина его ребра.

 

Дроби: правильная, неправильная, сравнение дробей
Правильная дробь

Дробь, числитель которой меньше знаменателя, называют правильной

Неправильная дробь

Дробь, числитель которой больше знаменателя или равен ему, называют неправильной.

Сравнение дробей
  • Из двух дробей с одинаковыми знаменателями больше та, числитель которой больше, и меньше та, числитель которой меньше.
  • Из двух дробей с одинаковыми числителями больше та, знаменатель которого меньше, и меньшая та, знаменатель которой больше.
  • Все правильные дроби меньше единицы, а неправильные — больше или равны единице.
  • Любая неправильная дробь больше любой правильной дроби.
Сложение и вычитание дробей с одинаковыми знаменателями
  • Чтобы найти сумму двух дробей с одинаковыми знаменателями, нужно сложить их числители, а знаменатель оставить тот же.
  • Чтобы найти разность двух дробей с одинаковыми знаменателями, надо из числителя уменьшаемого вычесть числитель вычитаемого, а знаменатель оставить тот же.
Сложение и вычитание смешанных чисел
  • Чтобы найти сумму двух смешанных чисел, надо отдельно сложить их целые и дробные части.
  • Чтобы найти разность двух смешанных чисел, надо от целой и дробной части уменьшаемого вычесть соответственно целую и дробную части вычитаемого.
Преобразование неправильной дроби в смешанное число

Чтобы неправильную дробь, числитель которой не делится нацело на знаменатель, преобразовать в смешанное число, нужно

  • числитель разделить на знаменатель;
  • полученное неполное частное записать как целую часть смешанного числа, а остаток — как числитель его дробной части.

227= смешанное число? 7322—211  227=317      

 

Преобразование смешанного числа в неправильную дробь

Чтобы преобразовать смешанное число в неправильную дробь нужно

  • целую часть числа умножить на знаменатель дробной части;
  • к полученному произведению прибавить числитель дробной части;
  • эту сумму записать как числитель неправильной дроби;
  • в его знаменателе записать знаменатель дробной части смешанного числа.

523= неправильная дробь?523=5*3+23=15+23=173

Десятичные дроби: свойства, сравнение, округление
Свойства десятичной дроби

Если к десятичной дроби справа приписать любое количество нулей, то получим дробь, равную данной.

Значение дроби, которая заканчивается нулями, не изменится, если последние нули в его записи отбросить.

2,23  = 2,230 = 2,230000005,50000=5,50000=5,5

Сравнение десятичных дробей

Из двух десятичных дробей больше та, у которой целая часть больше.

Чтобы сравнить две десятичные дроби с равными целыми частями и разным количеством цифр после запятой, надо

  • с помощью приписывания нулей справа уравнять количество цифр в дробных частях,
  • после чего сравнить полученные дроби поразрядно.

Сравнить 5,03 и 5,0375.5,03⏟2=5,0300⏟4    и     5,0375⏟4  ; 5,0300 < 5,0375.

Округление десятичных дробей

Для того чтобы десятичную дробь округлить до единиц, десятых, сотых и т. д., надо

  • все следующие за этим разрядом цифры отбросить.
  • если при этом первая из цифр, которые отбрасывают равна 0,1, 2, 3, 4, то последнюю из цифр, которые оставляют, не меняют;
  • если же первая из цифр, которые отбрасывют, равна 5, 6, 7, 8, 9, то последнюю из цифр, которые оставляют, увеличивают на единицу.

Округлить 5,248 и 3,952:а) до десятых:5,248≈5,2; 3,952≈4,0;б) до сотых:5,248≈5,25;3,952≈3,95.

Десятичные дроби: сложение, вычитание
Сложение десятичных дробей

Чтобы найти сумму двух десятичных дробей, нужно:

  •  уравнять количество цифр после запятых;
  •  записать слагаемые друг под другом так, чтобы каждый разряд второго слагаемого оказался под соответствующим разрядом первого слагаемого;
  •  сложить полученные числа так, как складывают натуральные числа;
  • поставить в полученной сумме запятую под запятыми.

Сложить 2,5 и 3,623.2,500⏟3 и 3,263⏟3;2,500+3,2635,763

Вычитание десятичных дробей

Чтобы найти разность двух десятичных дробей, нужно:

  •  уравнять количество цифр после запятых;
  • записать вычитаемое под уменьшаемым так, чтобы каждый разряд вычитаемого оказался под соответствующим разрядом уменьшаемого;
  •  выполнить вычитание так, как вычитают натуральные числа;
  • поставить в полученной разности запятую под запятыми.

Вычесть 3,27 и 3,009.3,270⏟3  и 3,009⏟3;3,270—3,0090,261

Десятичные дроби: умножение, деление
Умножение десятичных дробей

Чтобы перемножить две десятичные дроби, надо:

  • перемножить их как натуральные числа, не обращая внимания на запятые;
  • в полученном произведении отделить запятой справа столько цифр, сколько их стоит после запятых в обоих множителях вместе.

Умножить 1,5 и 2,25.2×2,2511,5+1125225·33,375 —количество цифр после запятой

Чтобы умножить десятичную дробь на 10, 100, 1000 и т. д., надо в этой дроби перенести запятую вправо на 1, 2, 3 и т. д. цифры.

Умножить 1,235 на 10, 100, 1000.а) на 10:1,235 ×10⏟1=12,35б) на 100:1,235 ×100⏟2 = 123,5в) на 1000:1,235 ×1000⏟3=1235,0 = 1235

Чтобы умножить десятичную дробь на 0,1; 0,01; 0,001 и т. д., надо в этой дроби перенести запятую влево соответственно на 1, 2, 3 и т. д. цифры.

Умножить 512,3 на 0,1,   0,01 и  0,001.а) на 0,1:512,3 ×0,1⏟1=51,23б) на 0,01:512,3 ×0,01⏟2=5,123в) на 0,001:512,3 ×0,001⏟3=0,5123

Деление десятичных дробей

Чтобы разделить десятичную дробь на десятичную, надо:

  • перенести в делимом и в делителе запятую вправо на столько цифр, сколько их содержится после запятой в делителе;
  • выполнить деление на натуральное число.

Разделить 24,2 на 0,02.24,2 : 0,02⏟ 2= 2420,0 : 2 = 2420 : 2 = 1210.

Чтобы разделить десятичную дробь на 10, 100, 1000 и т. д., надо в этой дроби перенести запятую влево на 1, 2, 3 и т. д. цифры.

 Разделить 25,5 на 10, 100, 1000.а)  на 10:25,5 : 10⏟1=2,55;б) на 100:25,5 : 100⏟2=0,255;в)  на 1000:25,5 : 1000⏟3=0,0255;

 

Среднее арифметическое

Средним арифметическим нескольких чисел называют результат деления сумму этих чисел на количество слагаемых.

Найти среднее арифметическое  чисел 15, 25 и 20.

15+25+20⏞сумма чисел3⏟количество чисел = 603= 20

Примечание:

Задача. Автомобиль 200 км ехал со скоростью 50 км/ч. Затем 120 км он ехал со скоростью 30 км/ч. Найти  среднюю скорость.

Здесь

 Vсредняя =Sобщtобщ .

1) 200 + 120 = 320(км) -весь путь;

2) 200 : 50 = 4(ч) — время, затраченное на 1-ую часть пути;

3) 120 : 30 = 4(ч) — время, затраченное на 2-ую часть пути;

4) 4 + 4 = 8(ч) — все время;

5) 320 : 8 = 40(км/ч) — средняя скорость.

Ответ: 40 км/ч.

Процент

Процентом  называют сотую часть величины или числа 1%=

Найти 4% от числа 20.20 : 100 = 0,2  (0,2 —это 1% от числа 20);0,2 × 4 =0,8( 0,8—искомое число).Или   4% = 4100 = 0,04;0,04 ×20 = 0,8.

Умножение и деление десятичных дробей 5 класс онлайн-подготовка на Ростелеком Лицей

Введение

 

Что такое запись числа в десятичной системе счисления? Например, запись числа 2536?

 

Это краткая запись суммы:

Десятичная дробь – это тоже краткая запись суммы:

С помощью запятой отделяется целая часть от дробной. Умножение или деление на 10, 100, 0,1, 0,01 и так далее не меняет цифры в записи, а лишь сдвигает запятую.

Напоминание:

 

Кроме того, стоит помнить, что любую десятичную дробь можно представить в виде произведения:

 

 

Умножение десятичной дроби на целое число или другую десятичную дробь

 

 

В предыдущем разделе были повторены все необходимые правила для того, чтобы умножать десятичную дробь на целое число или на другую десятичную дробь.

 

Пример: а) ; б)

а)

б)

Каждый раз запятая была убрана, но информация о ней сохранялась с помощью множителя 0,1, 0,01 и так далее. Потом было выполнено умножение целых чисел. И снова запятая возвращалась на место, учитывая множитель.

 

 

Правило умножения дробного числа на целое число

 

 

Чтобы умножить дробь на целое число, нужно умножать, не обращая внимания на запятую, затем в ответе вернуть запятую на место, то есть отделить ею столько же цифр в дробной части, сколько было в исходном числе.

 

Пример:

Выполняя умножение, не стоит обращать внимания на запятую.

Затем необходимо поставить запятую так, чтобы в дробной части получилось, как и раньше, 3 цифры.

 

 

Умножение двух десятичных дробей. Правило

 

 

Для умножения двух десятичных дробей принцип абсолютно такой же.

 

Правило

Чтобы перемножить две десятичные дроби, нужно их перемножить, не обращая внимания на запятые, затем в ответе отделить запятой столько цифр, сколько их было у обоих чисел вместе.

 

Пример 1

Сначала необходимо переписать каждую дробь в виде целого числа и вспомогательного множителя. Таким образом, каждая дробь будет представлена в виде произведения.

Затем нужно выполнить умножение целых чисел отдельно, вспомогательных множителей отдельно. Полученный результат нужно умножить на 0,001, то есть выполняется отделение запятой дробной части длиной в 3 цифры.

 

Пример 2

Нужно выполнить умножение, не обращая внимания на запятые. Количество цифр после запятой – 3, поэтому запятая ставится, отделяя 3 цифры. Последний ноль можно убрать из записи.

 

Пример 3

Выполняется умножение в столбик, при этом не обращая внимания на запятые, но помня, что в конце надо будет также отделить запятой 4 цифры.

 

 

Пример 4

В ходе урока мы уже выяснили, что умножать десятичные дроби технически означает просто умножать целые числа. Далее в ответе нужно отделить запятой знаков столько, сколько их было у всех чисел вместе.

Конечно, это правило распространяется и на случай нескольких множителей:

 

 

Деление десятичных дробей

 

 

Ситуация с делением десятичных дробей такая же: если уметь делить целые числа одно на другое, то тогда получится и десятичную дробь делить на другую десятичную дробь.

 

Пример

Когда заканчивается целое число, которое надо разделить, то ставится запятая и продолжается выполнение вычислений:

 

Пример

Здесь ситуация ровно такая же: как только кончается целая часть – ставится запятая:

То есть технически не важно, что делить на целое число – дробь или другое целое число. Алгоритм одинаковый.

 

 

Общий алгоритм деления

 

 

Пусть надо число  поделить на число . Это можно записать так:

 

 или 

Стоит помнить, что  и  можно умножить на одно и то же число, от этого результат не изменится.

В самом деле, можно делить , а можно , а можно . Можно все переписать в виде дробей. Результат все равно один – это равно 2.

 

 

Решение примеров

 

 

Пример. Одну десятичную дробь разделить на другую десятичную дробь:

 

Можно умножить обе части на одно и то же число. В данном случае это 10. Умножение на 10 означает сдвиг запятой вправо на одну позицию. Тогда второе число станет целым. А деление на целое число уже было изучено на этом уроке:

Задание: 1. ; 2.

1.     

2.     

 

Вывод

На этом уроке были изучены правила деления и умножения десятичных дробей. Кроме того, были также рассмотрены и решены различные примеры на данную тему.

 

Список литературы

1. Виленкин Н.Я.. Математика: учеб. для 5 кл. общеобр. учр. 17-е изд. – М.: Мнемозина, 2005.

2. Шевкин А.В. Текстовые задачи по математике: 5–6. – М.: Илекса, 2011.

3. Ершова А.П., Голобородько В.В. Вся школьная математика в самостоятельных и контрольных работах. Математика 5–6. – М.: Илекса, 2006.

4. Хлевнюк Н.Н., Иванова М.В.. Формирование вычислительных навыков на уроках математики. 5–9 классы. – М.: Илекса, 2011.          

 

Дополнительные рекомендованные ссылки на ресурсы сети Интернет

1. Портал «Школьный помощник» (Источник)

2. Портал «Школьная математика» (Источник)

3. Портал «МетаШкола» (Источник)

4. Портал Matematika-na.ru (Источник)

 

Домашнее задание

1. Вычислите:

а) ; б)

2. Вычислите:

а) ; б)

 

Умножение и деление натуральных чисел — правила и примеры для 5 класса » Kupuk.net

Одними из самых простых операций в математике являются умножение и деление натуральных чисел. В 5 классе после изучения арифметических действий школьников учат приёмам нахождения произведения и частного. Это знания, на которых базируется не только алгебра, геометрия, физика, химия, информатика, но даже и гуманитарные науки. Пожалуй, эти умения используются на практике больше любых других, полученных при обучении в средней школе.

Общие сведения

Математические вычисления сопровождают человека на всём протяжении его жизни. Когда произносится слово «число», имеется в виду определённый символ, определяющий количество чего-либо. Существуют различного вида выражения, например, целые, дробные, логарифмические. Но самыми простыми являются натуральные. Своё название они получили из-за применения в повседневной жизни. Их используют для счёта и определения порядка.

Таким образом, под натуральными числами понимают выражения, применяемые для определения количества любого физического объекта или присваивания порядкового номера. Например, 3, 1789, 9876, 100009. Если такие числа расположить в порядке увеличения, этот ряд называют натуральным. Последовательность 2, 3, 4, 5 будет именно такой. Нужно отметить, что натуральный ряд бесконечен, наибольшего значения в нём не существует.

Есть несколько систем счисления. В зависимости от неё, для обозначения используется различный набор символов. В России, США, европейских странах применяют арабскую систему. При этом в повседневности используется десятичная разрядность, то есть для записи чисел берут знаки от 0 до 9.

С числами можно выполнять любые действия. Их складывают, вычитают, перемножают и делят. Кроме этого, возводят в степень, извлекают из-под корня, логарифмируют и дифференцируют.

К основным свойствам натуральных чисел относят:

  • коммутативность при прибавлении;
  • бинарность операции умножения;
  • ассоциативность при сложении и умножении;
  • дистрибутивность произведения относительно сложения.

Эти свойства важны. На них часто опираются при решении примеров на умножение и деление в 5 классе средней школы. Каждая запись числа состоит из определённого количества разрядов. По сути, она составляет совокупность разрядных слагаемых. В качестве единиц принимают десятки. Любое натуральное выражение можно представить в виде суммы таких чисел. Например, 89 состоит из 8 десятков и 9 единиц. Значит, равенство 89 = 80 + 9 будет справедливым.

Неизвестную натуральную цифру принято обозначать маленькой латинской буквой эн (n). Интересно то, что пересчитать все числа невозможно.

Их количество бесконечно. Самое большое, которое удалось определить называется гугол. Оно содержит 100 нулей и является мерой атомов в физике.

Принцип умножения

Операция умножения подразумевает действие, заменяющее собой многократное сложение. Один из аргументов называют множимым, а другой множителем. Результатом умножения является произведение. Найти его довольно просто, если знать свойства операции.

К достаточным правилам, зная которые можно найти произведение любых чисел, относят:

  • Сочетательное — если при умножении произведения на любое число изменить порядок аргументов, результат не изменится. В буквенном виде закон имеет вид: a * b * c = a * c * b. Это правило можно доказать на опыте. Если взять квадраты размером 1 на 1 и построить из них блок 6 на 6, то фактически это будет перемножение 1 * 6 = 6. Полученный прямоугольник можно объединить с аналогичными 3. То есть 3 * 1 = 3. Общее число квадратов получится 1 * 6 * 3 = 18. Если же последовательность сборки изменить, сначала собрать предмет из трёх блоков, а потом к ним добавить 6, результат не изменится.
  • Распределительное — при выполнении действия над суммой и числом, можно отдельно каждый член выражения помножить на множитель, а затем результаты сложить. В математической записи правило выглядит так: a * (b + c) = a * b + a * c. По-другому операция называется раскрытием скобок. Это правило аналогично и для вычитания. Но при этом есть нюанс, что умножение выполняют сначала на уменьшаемое, а затем вычитаемое, и из первого вычитают второе.
  • Умножения на 0. Любое натуральное число при умножении на 0 даст в ответе 0. Справедливо и обратное утверждение.
  • Для умножения до 100 существует специальная таблица, которую необходимо знать наизусть.

    Следует также понимать, что при увеличении числа в десятки раз ответ увеличится на число нулей, стоящих в умножаемой цифре. Например, 34 * 10 = 340; 980 * 1000 = 980000. Так, выполняется сколь угодно сложное перемножение и для чисел большего десятка.

    Произведение часто находят методом «столбик». Суть способа состоит в том, что аргументы записывают один под одним. При этом самая правая цифра верхнего числа должна стоять над самой правой нижнего. Далее выполняют поразрядное умножение начиная с младших членов. Если при этом образуется высший разряд, он прибавляется к перемножаемому.

    Результат умножения следующего десятка сдвигается на единицу влево. Далее, складывают полученные результаты и получают искомое произведение.

    Выполнение деления

    Между нахождением частного и произведения существует тесная взаимосвязь. Особенно она просматривается при решении примеров на деление в 5 классе. По сути, эти 2 действия являются обратными друг другу. Математическим языком это можно описать как b * a = c → b = c / a. Эта зависимость в дальнейшем довольно сильно помогает решать сложные многозначные уравнения.

    Существует несколько способов поиска частного:

  • Последовательное вычитание. Нужно число разделить на другое. Чтобы найти ответ, понадобится из делимого вычитать делитель до тех пор, пока в ответе не получится 0. Затем следует подсчитать количество вычитаний. Это число и будет искомым ответом. На самом деле этот способ используется редко из-за своей громоздкости.
  • Представление в виде произведения. При решении примеров иногда удобно делимое разложить на множители, причём так, чтобы один из них легко можно было разделить на делитель. Например, 560 / 56 = (56 * 10) / 56 = 10.
  • Использование метода «уголок». Это наиболее часто применяемый способ. Делимое с делителем записывают в строчку, разделяя горизонтальной чертой. Вначале, сравнивая цифры, определяют неполное частное. Если в числе, что стоит справа, количество единиц меньше, добавляют следующий разряд. Затем подбирают такой множитель, чтобы при его умножении на делитель ответ не превышал выбранную часть делимого. Полученный результат записывают под низом делителя. Это будет первая цифра частного. Далее, от делимого вычитают результат умножения. Такие действия повторяют до тех пор, пока не получится 0.
  • Существуют методы, позволяющие проверить, насколько правильно найдено частное. Для этого нужно полученный ответ перемножить с делителем. Например, 12 / 4 = 3. Отсюда 3 * 4 = 12. Все три члена идентичные, значит, ответ найден верно.

    Следует знать, что есть приёмы, позволяющие облегчить выполнение действия. При нахождении результата деления, когда нужно найти частное двух одинаковых чисел, в ответе будет единица: 345/ 345 = 78 / 78 = 89976 / 89976 = 1.

    При этом 0, разделённый на любое число, даст в ответе 0. Делить же на него нельзя: выражение не будет иметь смысла.

    Решение примеров

    В 5 классе на математике всегда ученикам преподаватель предлагает решить определённые задания. Это нужно, чтобы школьник закрепил полученные теоретические знания и научился их применять на практике. Существуют сборники примеров по математике за 5 класс на умножение и деление для самостоятельной проработки. Прорешав успешно оттуда задачи, любой учащийся сможет утверждать, что он разобрался в теме.

    Вот некоторые из примеров, содержащиеся в таких задачниках:

  • Найти произведение выражения: 5 * 2 * (3 + 6) — 17. Вначале нужно выполнить операцию умножения, затем раскрыть скобки и от полученного результата отнять 17. Произведение пятёрки на двойку — это стандартное действие. Ответ операции нужно знать наизусть или сложить 2 раза цифру 5. Раскрыть скобки поможет распределительный закон. В итоге решение будет иметь следующий вид: 5 * 2 * (3 + 6) — 17 = 10 * (3 + 6) — 17 = 10 * 3 + 10 * 6 — 17 = 30 + 60 — 17 = 90 — 17 = 73.
  • Вычислить ответ: 450 :10 — 12 * 3 + 45: 45. Согласно правилам, сначала выполняют деление, а уже после вычитание и сложение. Определяя частное для первого члена, можно увидеть, что 450 = 45 * 10. В последнем же выражении число делится само на себя, значит, частное будет равно 1. Чтобы 12 умножить на 3, нужно сначала тройку перемножить с двойкой, а потом с единицей. Если это сделать, в ответе получится 36. Таким образом, решить пример можно так: 450: 10 — 12 * 3 + 45: 45 = 45 * 10: 10 — 12 * 3 + 1 = 45 — 12 * 3 + 1 = 45 — 36 + 1 = 45 — 37 = 8.
  • Решить уравнение 4 * n = 144. Исходя из смысла деления, можно записать n = 144: 4. Действие в столбик будет выглядеть так: 4 * 3 = 12, 3 пишется в частное, 14 — 12 = 2, сносится четвёрка и получается 24. Подбирается вторая цифра 4 * 6 = 24. Значит, в ответе получится n = 26.
  • С автобазы выехали 8 машин. В каждой из них было по 3 тонны груза. Каждая тонна размещалась в 42 ящиках. Сколько всего тары было отправлено со склада? Решение будет состоять из двух этапов. На первом нужно подсчитать, сколько ящиков было в каждом грузовике: 42 * 3 = 126. На втором определить число тары: 126 * 8 = 1008. Ответ нужно будет написать так: всего со склада было отправлено 1008 ящиков.
  • В начальных классах учителя при решении задач не разрешают пользоваться калькуляторами. Это необходимая мера.

    Ведь чтобы научиться, важно не только понимать суть действий, но и набраться необходимого опыта. При этом обязательно нужно наизусть выучить таблицу умножения.

    Онлайн уроки по математике за 5 класс